Download as pdf or txt
Download as pdf or txt
You are on page 1of 40

LEGALEDGE TEST SERIES

Part of the Most Comprehensive & Consistently Successful Study Material & Test Series Module, spanning across
both Offline and Online Programs in the entire Country. As a result LegalEdge was able to engineer Clean-Sweep-
Landslide figures of a handsome 64 Selections & 65 Selections in Top 100 (including AIR 1, 2 & 3 from Classroom
Contact Programs in 2022) & a whopping 273 selections & 327 selections in Top 500, in CLAT 2021 & CLAT 2022,
respectively. With AILET being no different, a total of 34 of our students found their way into NLU, Delhi in 2021 &
35 in 2022. In a nutshell, every second admit in a Top National Law School in 2021 & 2022 came from the LegalEdge
Preparation Ecosystem.

MOCK COMMON LAW ADMISSION TEST 2023


MOCK CLAT - 41

Scan this code after the test


Duration : 120 Minutes Candidate Name : _____________
Max. Marks : 150 Batch : _____________
Centre Name : __________ Contact No. : _____________

to punch in your answers


INSTRUCTIONS TO CANDIDATES
(Test ID: 2588465)
1. No clarification on the question paper can be sought. Answer the questions as they are.
2. There are 150 multiple choice objective type questions.
3. There is negative marking of 0.25 for every incorrect answer. Each question carries ONE mark. Total marks are
150
4. You have to indicate the correct answer by darkening one of the four responses provided, with a BALL PEN
(BLUE OR BLACK) in the OMR Answer Sheet.
Example: For the question, "Where is the Taj Mahal located?", the correct answer is (b).
The student has to darken the corresponding circle as indicated below:
(a) Kolkata (b) Agra (c) Bhopal (d) Delhi
Right Method Wrong Methods

5. Answering the questions by any method other than the method indicated above shall be considered incorrect and
no marks will be awarded for the same.
6. More than one response to a question shall be counted as wrong.
7. Do not write anything on the OMR Answer Sheet other than the details required and, in the spaces, provided for.
8. You are not required to submit the OMR Answer Sheet and Test Paper after the test.
9. The use of any unfair means by any candidate shall result in the cancellation of his/her candidature.
10. Impersonation is an offence and the student, apart from disqualification, may have to face criminal prosecution.
11. You have to scan the QR code only after completion of offline test.
12. You cannot leave the examination hall without punching your answers on the portal.
SECTION-A: ENGLISH LANGUAGE

Directions (Q.1-Q.30): Read the following passage carefully and answer the questions that follow.

Passage (Q.1-Q.5): When Pakistan came into existence in 1947, it was the realisation of a yearning by a section
of India’s Muslims, who feared domination by the Hindu majority in a postcolonial India. The tensions between
Hindus and Muslims on the Indian subcontinent were recognized by Great Britain, and the British made their
final plans to surrender the “Jewel in the Crown” of their empire after 90 years—their “Two Nations Theory”—
premised on the notion of a separate homeland for the subcontinent’s Muslim minority. This evolved into a
collective vision championed by Muslims, who broke ranks from their nationalist counterparts of all
backgrounds. The partition of the Indian subcontinent at the time of independence from colonial rule in August
1947 entailed the creation of two new sovereign states—India and Pakistan, with the latter composed of areas
where Muslims were in a numerical majority. Partition involved not only the institution of separate
administrative and political structures for the two newly independent states but also a momentous upheaval of
population migration, as well as unanticipated bloodshed and brutality among Hindus and Muslims on an
unimaginable and inexplicable scale. Twelve to 14 million people were caught up in this process of mass
migration; more than a million were killed in violent encounters, and an estimated 75,000 women were abducted
and subjected to sexual violence (Butalia, 2000). In 1946 and 1947, the subcontinent was torn by Hindu–Muslim
riots in an atmosphere of religious hatred inflamed by the emerging reality of partition (Gooptu, 2002).

The partition of India and Pakistan had separate meanings and identities for both countries. As Gooptu (2002)
further postulates, on the Indian side, partition was portrayed as an aberration in India’s heroic struggle for
independence and her triumphant march to nationhood; the partition was projected as the product of an act of
betrayal of the national cause by so-called separatist Muslims. On the Pakistani side, partition was seen as a
necessary step toward the realisation of nationhood and a safe homeland for South Asian Muslims. The lived
experience of partition and the trauma of death and destruction, hardship and dislocation, misery and debasement
has been shrouded in what some have recently called a tyranny of silence. The history of loss, suffering, and
uprootedness, which is how the people of the subcontinent actually experienced partition, was hardly ever aired
in public in the first 40 years following the partition. Collective amnesia and denial appear to have been the ways
in which this catastrophic experience of violence and displacement was psychologically and culturally
negotiated.

After independence, the decision as to what system of government the new nation should adopt was never fully
reached. In the decades following independence, this irresolution contributed to a recurrent pattern of crisis:
repeated coups and extended periods during which martial law replaced civilian government, violent deaths of
several national leaders, periodic strife among ethnic groups, and, most traumatically, a civil war in 1971
involving linguistic and cultural issues that divided the East and West parts of the country in two. After India
intervened, East Pakistan seceded and renamed itself Bangladesh.

1. According to the passage, what was the cause of the partition between India and Pakistan?
(a) This was a result to satiate the egoistic nature of the British parliamentarians of that period.
(b) The tensions between Hindus and Muslims were realised by the British, and this resulted in the partition.
(c) The violent protests throughout the country made partition undeniable by the then British government.
(d) The clashes between Hindus and Muslims in the name of religion coerced the British to divide India.

2. Which among the following best depicts the different ways in which India and Pakistan perceived the partition,
respectively?
(a) Result of the struggle for independence; Realisation of a safe homeland.
(b) March to nationhood; Freedom from religious bonding.
(c) Act of betrayal; necessary step for a safe homeland.
(d) Struggle for the cause of independence; struggle to remain dependent.

Head Office: 127, Zone II, MP Nagar, Bhopal |+91-7676564400| https://www.toprankers.com Page 2 of 40
3. Out of the following, the author of the passage is most likely to agree with which among the following?
(a) The “Two Nations Theory” of the British government is overrated and emphasised.
(b) Military coups were common side in the divided countries: India and Pakistan.
(c) The partition of undivided India was perceived differently by the two countries.
(d) Both a and c are correct.

4. Select the correct synonym of the term ‘Upheaval’.


(a) Pandemonium (b) Armistice (c) Ceasefire (d) Natatorium

5. Which of the following is not supported by the passage?


(a) The mayhem following the partition was broadcast extensively for the first forty years.
(b) The “Two Nations Theory” evolved into a collective vision championed by Muslims.
(c) Muslims had the largest population in the Pakistan region of undivided India.
(d) The irresolution of which system of governance to follow in Pakistan contributed to a recurrent pattern of
crisis.

Passage (Q.6-Q.10): Something has gone wrong with the internet. Even Mark Zuckerberg knows it. Testifying
before Congress, the Facebook CEO ticked off a list of everything his platform has screwed up, from fake news
and foreign meddling in the 2016 election to hate speech and data privacy. “We didn’t take a broad enough view
of our responsibility,” he confessed. There have always been outsiders who criticised the tech industry — even
if their concerns have been drowned out by the oohs and aahs of consumers, investors, and journalists.

But today, the most dire warnings are coming from the heart of Silicon Valley itself. The man who oversaw the
creation of the original iPhone believes the device he helped build is too addictive. Even the inventor of the
World Wide Web fears his creation is being “weaponized.” To understand what went wrong — how the Silicon
Valley dream of building a networked utopia turned into a globalised strip-mall casino overrun by pop-up ads
and cyberbullies and Vladimir Putin — we spoke to more than a dozen architects of our digital present. If the
tech industry likes to assume the trappings of a religion, complete with a quasi-messianic story of progress, the
Church of Tech is now giving rise to a new sect of apostates, feverishly confessing their own sins. And the
internet’s original sin, as these programmers and investors and CEOs make clear, was its business model. To
keep the internet free, the technological elite needed something to attract billions of users to the ads they were
selling. And that something, it turns out, was outrage. The advertising model of the internet was different from
anything that came before. Whatever you might say about broadcast advertising, it drew you into a kind of
community, even if it was a community of consumers. The culture of the social media era, by contrast, doesn’t
draw you anywhere. It meets you exactly where you are, with your preferences and prejudices — at least as best
as an algorithm can intuit them. “Microtargeting” is nothing more than a fancy term for social atomization — a
business logic that promises community while promoting its opposite.

Why, over the past year, has Silicon Valley begun to regret the foundational elements of its own success? The
obvious answer is November 8, 2016. For all that he represented a contravention of its lofty ideals, Donald
Trump was elected, in no small part, by the internet itself. What we’re left with are increasingly divided
populations of resentful users, now joined in their collective outrage by Silicon Valley visionaries no longer in
control of the platforms they built. The unregulated, quasi-autonomous, imperial scale of the big tech companies
multiplies any rational fears about them — and also makes it harder to figure out an effective remedy. Could a
subscription model reorient the internet’s incentives, valuing user experience over ad-driven outrage? Or should
we break up these new monopolies entirely in the hope that fostering more competition would give consumers
more options?

Head Office: 127, Zone II, MP Nagar, Bhopal |+91-7676564400| https://www.toprankers.com Page 3 of 40
Silicon Valley, it turns out, won’t save the world. But those who built the internet have provided us with a clear
and disturbing account of why everything went so wrong — how the technology they created has been used to
undermine the very aspects of a free society that made that technology possible in the first place.
A set of questions accompanies the passage below. Choose the best answer.

6. The author makes all of the following arguments in the passage EXCEPT:
(a) Today, the outsiders are not alone in warning the globe about the internet.
(b) There was something fundamentally wrong with the idea of the internet itself.
(c) There is an underlying difference between broadcast and social media advertising.
(d) The internet played no less than a substantive role in Donald Trump’s election.

7. Which of the following sets of words and phrases serves best as keywords of the passage?
(a) Internet, Privacy, Advertisements
(b) Utopia, Globalization, Downfall
(c) Donald Trump, Warnings, Technology
(d) Internet, Business Model, Regret

8. Which of the following statements best summarizes the central point of the passage?
(a) Once the internet's foundational elements, keeping it free would result in its downfall.
(b) Even those who designed our digital world are aghast at what they created; a breakdown of what went wrong
— from the architects who built it.
(c) The technology that created the internet has been misused to make the internet a worse place than before.
(d) Advertisements and micro-targeting have led the internet to foster technology spaces it shouldn’t have
enabled.

9. But those who built the internet have provided us with a clear and disturbing account of why everything went so
wrong — how the technology they created has been used to undermine the very aspects of a free society that
made that technology possible in the first place.’ Which of the following literary devices is used in the following
argument?
(a) Paradox. (b) Metaphor. (c) Juxtaposition. (d) Hyperbole.

10. Which of the following is the tone of the passage?


(a) Contemplative (b) Sarcastic (c) Effervescent (d) Analytical

Passage (Q.11-Q.14): The past two and a half years have been extraordinary. What we are seeing is surely more
than the progression of just another business cycle. There is an economic crisis looming. The unnerving
combination of a global pandemic compounded by energy scarcity, rapid inflation, and geopolitical tensions
boiling over has people wondering what certainties are left. Today’s events might even feel like a cluster of
earthquakes that is reshaping our world.

We have been here before. Similar “earthquakes” have struck the past: in the immediate aftermath of World
War II (1944–46), during the period around the oil crisis (1971–73), and at the time of the breakup of the Soviet
Union (1989–92). Like a real earthquake, each of them changed the global landscape with the sudden release of
powerful underlying forces that had been building up around a fault line over time—but in these cases, unfolding
over a few years rather than in a big bang. Each of them ushered in a new era: the Postwar Boom (1944–71), the
Era of Contention (1971–89), and the Era of Markets (1989–2019). Are we now on the cusp of a new era presaged
by today’s earthquakes?

A new paper from the McKinsey Global Institute suggests a framework to imagine the new era, drawn from a
historical perspective of the structural tectonics that underpinned the world we have today and how they might

Head Office: 127, Zone II, MP Nagar, Bhopal |+91-7676564400| https://www.toprankers.com Page 4 of 40
play out in the next era. Working out how to respond to the current moment and the path ahead is complex and
requires boldness.

We are reminded most of the aftermath of the oil shocks in the early 1970s, which shared features resonant with
today: an energy crisis, a negative supply shock, the return of inflation, a new monetary era, rising multipolar
geopolitical assertion, resource competition, and slowing productivity in the West. The aftershocks came in many
waves and took almost 20 years to resolve. The return of stability required investment in energy independence
by non-OPEC countries and painful monetary stabilization. In addition, there was strong political will,
personified by Ronald Reagan, Margaret Thatcher, and Deng Xiaoping.

But there are differences between now and the earthquake of the early 1970s that arguably magnify the reasons
for concern. Today’s world is much more globally entwined, financially leveraged, and carbon constrained. This
time, can we do better and write a new narrative of progress more quickly?

Of course, we could be overblowing the momentousness of current events. However, this is different from other
tremors like the Asian financial crisis in 1997, the dot-com bust in 2000, and the global financial crisis in 2008.
Most of these events were on the demand side and were largely contained in a region or a sector. Today, however,
we face a supply-side crisis, inherently physical rather than psychological, against a backdrop of a shifting
geopolitical landscape upon which the crisis needs to be resolved.

Moreover, today’s earthquakes have largely come as surprises, shaking the world after a 30-year era of relative
calm. In truth, our professional lives have played out on one clear and consistent global landscape—one where
perhaps we have embedded many implicit assumptions and beliefs about how the world works, which are now
under direct challenge.
A set of questions accompanies the passage below. Choose the best answer.

11. The author of the passage makes which of the following arguments in the passage?
(a) The next era needs to be started from a fundamentally different point from the previous one.
(b) Today’s economic crises have come after a relatively long time, so this is not a surprise.
(c) The world is taking the present day’s economic crises quite casually, which may make it pay.
(d) None of the above.

12. Of the following sets of words, identify the set that is closest to the concerns of the passage.
(a) Earthquakes, World War, Disaster
(b) “Earthquakes”, Similarities, Differences
(c) Economic crisis, Challenges, Remedies
(d) Today’s earthquakes, Past, Future
13. Which one of the following best expresses the difference between the “Earthquakes” of the present and the past?
(a) The significance of past events is far blown out of proportion than that of todays.
(b) The effects of the past “earthquakes” were more concentric than today’s scattered effects.
(c) The present day “earthquakes” have largely come as surprises since they have come after a considerable time
gap.
(d) The resources to fight against today’s “earthquakes” are obscurely less than the “earthquakes” of the past.
14. The information in the passage allows for which of the following to be inferred?
(a) There is a possibility that the present day earthquakes are perceived to be greater than they actually are.
(b) The previous “earthquakes” used to develop over a long period of time, culminating in a catastrophe.
(c) The paper cited in the passage from the McKinsey Global Institute suggests that the previous economic crises
were ill-managed.
(d) The world of the past was much more financially leveraging, carbon constrained, and globally intertwined
than today’s world.
Head Office: 127, Zone II, MP Nagar, Bhopal |+91-7676564400| https://www.toprankers.com Page 5 of 40
Passage (Q.15-Q.19): Passage: The human intellect is said to be so constituted that general ideas arise by
abstraction from particular observations and therefore come after them at a point in time. If this is what actually
occurs, as happens in the case of a man who has to depend solely upon his own experience for what he learns —
who has no teacher and no book — such a man knows quite well which of his particular observations belong to
and are represented by each of his general ideas. He has a perfect acquaintance with both sides of his experience,
and accordingly, he treats everything that comes in his way from a right standpoint. This might be called the
natural method of education.

Contrarily, the artificial method is to hear what other people say, to learn and to read, and so to get your head
crammed full of general ideas before you have any sort of extended acquaintance with the world as it is, and as
you may see it for yourself. You will be told that the particular observations which go to make these general
ideas will come to you later on in the course of experience; but until that time arrives, you apply your general
ideas wrongly, you judge men and things from a wrong standpoint, you see them in a wrong light, and treat them
in a wrong way. So it is that education perverts the mind.

This explains why it so frequently happens that, after a long course of learning and reading, we enter upon the
world in our youth, partly with an artless ignorance of things, partly with wrong notions about them; so that our
demeanour savours at one moment of nervous anxiety, at another of mistaken confidence. The reason for this
is simply that our head is full of general ideas which we are now trying to turn to some use, but which we hardly
ever apply rightly. This is the result of acting in direct opposition to the natural development of the mind by
obtaining general ideas first, and particular observations last: it is putting the cart before the horse. Instead of
developing the child’s own faculties of discernment, and teaching it to judge and think for itself, the teacher uses
all his energies to stuff its head full of the ready-made thoughts of other people. The mistaken views of life,
which spring from a false application of general ideas, have afterwards to be corrected by long years of
experience; and it is seldom that they are wholly corrected. This is why so few men of learning are possessed of
common-sense, such as is often to be met with in people who have had no instruction at all.

15. What is the main idea of the passage?


(a) The abstraction of one’s general ideas should follow from and be rooted in direct experience and observation.
(b) The artificial method of education is essentially the inverse of the natural method.
(c) Artificial education produces at once a nervous anxiety and a mistaken confidence in all.
(d) Common sense is uncommon among learned men, women and living beings.

16. The author contends that artificial method of education leads to “artless ignorance” in the third paragraph to
point out that:
(a) We see the world around us as per the ready-made thoughts of other people rather than trying to see and
think for ourselves.
(b) We intend to be generally ignoramus about the art and beauty around us.
(c) Wrong general ideas are filled into our minds and must be corrected by therapy.
(d) People who have no instructions at all are not ignorant and have better common sense.

17. In the author’s view, education perverts the mind because


(a) We judge men and things from the wrong standpoint.
(b) We are filled with general ideas but have no practical observations.
(c) We apply cognition precisely where it is required.
(d) We treat men and animals in the wrong way.

Head Office: 127, Zone II, MP Nagar, Bhopal |+91-7676564400| https://www.toprankers.com Page 6 of 40
18. What is the meaning of the idiom “putting the cart before the horse” in the passage?
(a) Reverse the proper order or procedure of something.
(b) Irreversible order or procedure of something.
(c) To order the cart before the horse
(d) To order something in its proper sequence.

19. Which of the following is most SIMILAR to the word “demeanour”?


(a) Gesture (b) Posture (c) Conduct (d) Callousness

Passage (Q.20-Q.24): Finance and financial practices have become increasingly significant to capital
accumulation in developing countries (Karwowski and Stockhammer, 2017). There is growing literature on these
practices under the discussion of financialisation. This article examines the modalities of financialisation in India
through the case study of the telecom sector. This sectoral study helps illustrate the unique features of
financialisation in India and contrasts it with those in other developing countries.

Financialisation is a conceptually complex and contested phenomenon. It is associated with a range of meanings,
methods and theories focussing on ‘the increasing role of financial motives, financial markets, financial actors
and financial institutions in the operation of domestic and international economies’ (Epstein, 2005, p. 3). This
article adopts Krippner’s general definition of financialisation as ‘a pattern of accumulation, in which profits
accrue primarily through financial channels rather than through trade and commodity production’ (2011, p. 174).
Drawing upon this definition, the article shows how the role and function of finance in India changed after the
neoliberal reforms of the 1990s.

Telecommunication is one of India’s most successful sectors: starting from a backlog of under-provision and
poor services, mobile telephony has boomed since it was opened up to private investment in 1994, and currently
provides efficient services to the vast majority of Indians at affordable rates. Using the Indian telecom sector as
a case study, this article demonstrates how the function of finance changed rapidly, from facilitating business
operations to making telecom companies investable financial assets that could be bought and sold for profit. This
sector was chosen for three main reasons: first, its oligopolistic market structure provided (____) into how big
private businesses, with their substantial financial resources and political and economic networks interacted with
each other and the various business practices they (______) to maximise the value of their telecom companies.
Second, the telecoms sector is regulation-driven, as spectrum rights which are at the heart of the
telecommunications service, provision are both created and regulated by the state. The state and business must
constantly interact, as the state is deeply involved at every level, in the production, financing, distribution and
consumption of telecommunications services in India. Such interactions provided an excellent opportunity to
study the role that the state played in driving, underwriting and managing the process of financialisation. Finally,
the telecom sector in India was developed and financialised over a relatively short period from 1994 onwards,
and the developments in the sector are remarkably well documented. This article aims to contribute to the
literature on financialisation in developing countries and more specifically, on India as there is no systematic
empirical study of how (if at all) the process of financialisation has evolved in India, and there are no sectoral
studies of financialisation in specific industries.

After this introduction, this article examines the challenges to theorising financialisation and applying it to
developing countries. Then it provides an empirical analysis of the Indian telecom sector (between 1994 and
2019) and details how the process of financialisation takes place. Finally, the article compares the features and
trends of financialisation in India to those in developing countries before concluding.

Head Office: 127, Zone II, MP Nagar, Bhopal |+91-7676564400| https://www.toprankers.com Page 7 of 40
20. The author would disagree with which of the following?
(a) This article adopts Krippner’s general definition of financialisation as ‘a pattern of accumulation, in which
profits accrue primarily through financial channels rather than through trade and commodity production’.
(b) Drawing upon this definition, the article shows how the role and function of finance in India changed after
the neoliberal reforms of the 1890s.
(c) Financialisation is a conceptually complex and contested phenomenon.
(d) Financialisation is associated with a range of meanings, methods and theories focussing on ‘the increasing
role of financial motives, financial markets, financial actors and financial institutions in the operation of
domestic and international economies’.

21. ‘This sector was chosen for three main reasons: first, its oligopolistic market structure provided (_______) into
how big private businesses, with their substantial financial resources and political and economic networks,
interacted with each other and the various business practices they (________) to maximise the value of their
telecom companies. Fill in the blanks with the correct option.
(a) Foresight; undertaken (b) Insight; undertook
(c) Hindsight; took (d) Sight; partaken

22. The telecoms sector is regulation-driven, as spectrum rights which are at the heart of the telecommunications
service provision are both created and regulated by which of the following?
(a) The private business owners (b) The consumers
(c) The state (d) The benefactors

23. Which of the following is/are true?


Statement 1: Telecommunication is one of India’s most successful sectors.
Statement 2: Telecommunication services currently provide efficient services to the vast majority of Indians at
affordable rates.
Choose the correct code:
(a) Statement 1 only (b) Statement 2 only
(c) Neither Statement 1 nor 2 (d) Both statements 1 and 2

24. Which of the following best reflects the title of the passage?
(a) Finance and financial practices in developed countries.
(b) Financialisation in India through the case study of the telecom sector.
(c) The unique features of financialisation in India and its contrasts with other developing countries.
(d) Growing literature of practices under the discussion of financialisation.

Passage (Q.25-Q.30): Climate sceptics are winning the argument with the public over global warming, the
world’s most celebrated climate scientist, James Hansen of NASA, said in London yesterday.

“It is happening even though climate science itself is becoming ever clearer in showing that the earth is in
increasing danger from rising temperatures”, said Dr. Hansen, who heads NASA’s Goddard Institute of Space
Studies, and is widely thought of as “the father of global warming” - his dramatic alert about climate change in
US Senate hearings in July 1988 put the issue on the world agenda.

Since then, he has been one of the most outspoken advocates of drastic climate action, and yesterday he also
publicly criticised Germany’s recent decision to abandon its new nuclear power programme, formerly a key part
of German climate measures, in the wake of the Fukushima nuclear disaster in Japan earlier this year.

Head Office: 127, Zone II, MP Nagar, Bhopal |+91-7676564400| https://www.toprankers.com Page 8 of 40
“I think it was a big mistake,” he said. “And I think the Prime Minister [German Chancellor Angela Merkel]
knows that, as she’s a physicist, but I think the political reality is she couldn’t stay in office if she expressed that
opinion.”

In a briefing at the Royal Society, Dr. Hansen, pictured, was frank about the success of public opinion of what
he termed “the climate contrarians”, in effectively lessening public concern about global warming. He said:
“They have been winning the argument for several years, even though the science has become clearer.”

“There’s been a very strong campaign by those who want to continue fossil fuel ‘business as usual’, and the
scientific story has not been powerful enough to offset that push.”

Part of the problem, he said, was that the climate sceptic lobby employed communications professionals, whereas
“scientists are just barely competent at communicating with the public and don’t have the wherewithal to do it.”

The result was, he said, that in recent years “a gap has opened between what is understood about global warming
by the relevant scientific community, and what’s known by the people who need to know - and that’s the public.
However, there’s nothing that has happened to reduce our scientific conclusion that we are pushing the system
into very dangerous territory; in fact, that conclusion has become stronger over that same time period.”

Asked if anything might re-alert the public to the dangers of climate change, Dr. Hansen said: “Mother Nature.”

Significant climatic extreme events were now occurring over 10 to 15 per cent of the planet annually, whereas
between 1950 to 1980; they occurred over less than 1 per cent. He added: “So in places like Texas this year,
Moscow last year, and Europe in 2003, the climate change is so big that they are undeniable. Within 10 to 15
years, they’re going to occur over 15 to 20 per cent of the planet. So, people have to notice that the climate is
changing.”

Northern Europe, 2003 Shrivelled French grapes at the end of Europe’s hottest summer on record in 2003. The
heatwave led to health crises in several countries, and more than 40,000 people are thought to have died. Britain
experienced its first (and so far only) 100+ F air temperature - 101.3°F (38.5°C) recorded at Brogdale, Kent, on
10 August.

25. Why did Dr. James Hansen criticize Germany?


(a) Dr. James Hansen criticized Germany because it had denied making contributions to the fund meant for
combating the menace of global Warming.
(b) Dr. James Hansen criticized Germany because it decided to withdraw its membership from the Paris
Agreement.
(c) Dr. James Hansen criticized Germany because it said it could not decrease the level of carbon emissions as
it would hurt its industrial sector.
(d) Dr. James Hansen criticized Germany because it has decided to discard its new nuclear power programme,
formerly a key part of German climate measures.

26. According to Dr. James Hansen, what is/are the reasons behind the extensive use of fossil fuels, even when their
use is detrimental to the environment?
(a) People are extensively using fossil fuels because they are not aware of the ill effects of using them.
(b) The use of fossil fuels is going on with full fling because those who want to continue using them, campaign
strongly in favour of it.
(c) According to Dr. Hansen, the scientific community lacks the ability to keep forth its arguments effectively.
(d) Both (b) and (c).

Head Office: 127, Zone II, MP Nagar, Bhopal |+91-7676564400| https://www.toprankers.com Page 9 of 40
27. As per the given passage, the German Chancellor Angela Merkel belongs to which of the following science
fields?
(a) She is a botanist. (b) She is an environmentalist.
(c) She is a physicist. (d) She is a mathematician.

28. Which of the following places, as mentioned in the passage, has experienced a temperature above 100°F for the
first time?
(a) Texas (b) Moscow (c) France (d) Britain

29. Given below is a sentence from the passage. It might have an error. If the sentence contains an error, the part
containing the error is your answer.
Significant climatic “extreme events” are now (A)/ occurring over 10 to 15 per cent of (B) the planet annually,
whereas between 1950 to 1980, (C)/they occurred over less than 1 per cent.(D)
(a) B (b) C (c) A (d) D

30. Whom has Dr. Hansen referred to as “the climate contrarians”, as given in the passage?
(a) The scientific community that is unable to make people aware of the real scenario.
(b) The politicians/leaders of the developed nations who are unwilling to give up their conventional models of
growth and are ready to ignore the environmental issues.
(c) The general public who is not aware of the harmful effects of fossil fuels and is using them injudiciously.
(d) The people who are endorsing the use of conventional resources and minimizing the carefulness of people
about global warming.

Head Office: 127, Zone II, MP Nagar, Bhopal |+91-7676564400| https://www.toprankers.com Page 10 of 40
SECTION-B : CURRENT AFFAIRS, INCLUDING GENERAL KNOWLEDGE

Directions (Q.31–Q.65): Read the information given below and answer the questions based on it.

Passage(Q.31-Q.35): On October [X], the United Nations Charter came into force. Thus, after the six years of
the Second World War, the United Nations Organization was born, with the goals of safeguarding international
peace and security and establishing broader cooperation among nations.

Each year, October 24 marks United Nations Day. This celebration is also intended to highlight the
accomplishments of the Organization and to anticipate the future challenges it will face, particularly in the
context of the post-covid-19 economic crisis.
Its actions have been rewarded with 12 Nobel Peace Prizes, the last of which was awarded to the World Food
Programme in 2020.

The UN is therefore today a doubly universal organization: it offers a space for dialogue for all the states of the
world and for all subjects. However, it is also subject to numerous criticisms and is being questioned: in
particular, the rise of nationalism rejecting international cooperation - which is the basis of the UN method of
action - threatens the legitimacy and scope of the Organization’s action.

31. The United Nations (UN) is an international organization founded in ______. It is currently made up of _______
Member States.
(a) 1950, 186 (b) 1945, 193 (c) 1956, 174 (d) 1948, 162

32. What is the Significance of India at UN? Choose the correct options from the following –
I) India is one of the founding members of the UN. India has deployed a lot of soldiers since years and is an
active member of UN Peacekeeping Forces in the conflicted zones.
II) Since its independence and even before that, India has been an active participant in all initiatives undertaken
by the UN like Millennium Development Goals, Sustainable development goals and various UN summits,
including on climate change.
(a) Only I is correct (b) Only II is correct
(c) Either I or II is correct (d) All of the above

33. What are the UN’s Contributions till date? Choose the incorrect options from the following –
I) Increment in the UN membership
II) Decolonisation
III) Cooperation with Militias
IV) Peacekeeping
(a) Only I (b) Both II & IV (c) I,II & IV (d) Only III

34. At the time of formation, UN had how many permanent members?


(a) 5 (b) 10 (c) 8 (d) 3

35. What are the UN's Major Setbacks? Choose the correct statements from the following –
I) The organisation had not been able to cope up with the increasing globalisation.
II) The UN has been a laggard in overall development; no institutional arrangement is there to deal in particular
with Pandemics or new technologies such as Artificial Intelligence.
III) The UN has been unable to present itself as a bipolar and multilateral organisation.
(a) Only I is correct (b) Both I & II are correct
(c) Both II & III are correct (d) All of the above

Head Office: 127, Zone II, MP Nagar, Bhopal |+91-7676564400| https://www.toprankers.com Page 11 of 40
Passage(Q.36-Q.40): On May 26, 1999, at a time the Indian economy was opening up to global markets, ISRO
for the first time launched a commercial satellite — its workhorse, the Polar Satellite Launch Vehicle (PSLV),
put a 45-kg German satellite and a 110-kg Korean satellite into desired orbits. That was a red-letter day for the
Indian space programme. And since then, the PSLV, which has been the mainstay of launch services provided
by ISRO, has launched another 343 foreign satellites in commercial missions.

More than 23 years and 345 foreign satellite launches later, as a depression formed over the Bay of Bengal
keeping the weatherman and disaster management units from multiple state governments on their toes, India’s
Geosynchronous Satellite Launch Vehicle-Mark3 (GSLV-Mk3) or LVM3 as ISRO now calls it, soared the night
sky in its maiden commercial mission.

36. The Indian Space Research Organisation’s (ISRO) heaviest rocket Launch Vehicle Mark 3 (LVM3 or GSLV
Mark 3) has successfully orbited _____ satellites of U.K.-based OneWeb.
(a) 36 (b) 20 (c) 30 (d) 42

37. What is LVM 3? Choose the correct statements based on it –


I) The LVM3-M2 mission is a dedicated commercial mission for a foreign customer OneWeb, through
NewSpace India Limited (NSIL), a Central Public Sector Enterprise (CPSE).
II) This newest rocket is capable of launching 4,000-kilogram class of satellites into GTO (Geosynchronous
Transfer Orbit) and 8,000 kgs of payloads into LEO.
III) It is a three-stage launch vehicle consisting of two solid propellant S400 strap-ons on its sides and core stage
comprising L210 liquid stage and C35 cryogenic stage.
(a) Only I is correct (b) Both I & II are correct
(c) Both II & III are correct (d) All of the above

38. What are the features of LVM 3? Choose the correct options –
I) First Indian rocket with eight-ton payload
II) First NSIL Mission with LVM3
III) First launch of LVM3 to LEO
(a) Only III is correct (b) Both I & II are correct
(c) Both II & III are correct (d) All of the above

39. What is OneWeb Constellation? Choose the incorrect options from the following –
I) OneWeb Constellation operates in a LEO Polar Orbit Satellites are arranged in 14 rings (Orbital planes) with
50 satellites in each plane.
II) The orbital planes are 1200 km above the Earth. Each satellite completes a full trip around the earth every
93 minutes.
(a) Only I (b) Only II
(c) None of the above (d) All of the above

40. Polar Satellite Launch Vehicle (PSLV) first launch was in which year?
(a) 1998 (b) 1994 (c) 1995 (d) 1990

Passage(Q.41-Q.45): Prime Minister Narendra Modi on Tuesday evening reviewed the construction of the
National Maritime Heritage Complex (NMHC) site at Gujarat’s Lothal via video conferencing. “There are many
such tales of our history that have been forgotten,” the PM said. “Lothal was not only a major trading centre of
the Indus Valley Civilisation, but it was also a symbol of India’s maritime power and prosperity.”

Indian archaeologists started the search for cities of the Harappan Civilisation post-1947 in Gujarat’s Saurashtra.
Archaeologist SR Rao led the team which discovered a number of Harappan sites at the time.

Head Office: 127, Zone II, MP Nagar, Bhopal |+91-7676564400| https://www.toprankers.com Page 12 of 40
In later excavations, ASI unearthed a mound, a township, a marketplace, and the dock. Adjacent to the excavated
areas stands the archaeological site museum, where some of the most prominent collections of Indus-era
antiquities in India are displayed.

41. Name the world’s oldest dockyard?


(a) Portsmouth Historic Dockyard (b) Hindustan Shipyard Limited
(c) Lothal Dockyard (d) Royal Naval Dockyards

42. What is the National Maritime Heritage Complex? Choose the correct options from the following –
I) It has three theme parks – Memorial theme park, Maritime & Navy theme park and Climate theme park.
II) The NMHC is being developed with the aim of displaying India’s diverse maritime heritage and also help
Lothal emerge as a world-class international tourist destination.
III) It will act as a center for learning and understanding India’s maritime history.
(a) Only I is correct (b) Both I & II are correct
(c) Both II & III are correct (d) None of the above

43. Lothal was a thriving trade center in ancient times, with its trade of_________, __________ reaching West Asia
and ________.
(a) Spices, Rice, Saudi Arabia (b) Beads, gems & ornaments, Africa
(c) Cotton, Silk & Velvet, Iran (d) Sugarcane, Fish, Australia

44. What is the Heritage Value of Lothal? Choose the incorrect options –
I) Lothal was nominated in April 2014 as a UNESCO World Heritage Site, and its application is pending on
the tentative list of UNESCO.
II) The excavated site of Lothal is the only port-town of the Indus Valley Civilisation.
III) In the region, it can be compared with other Indus port towns of Balakot (Pakistan), Khirasa (in Gujarat’s
Kutch) and Kuntasi (in Rajkot).
(a) Only II (b) Both II & III
(c) All of the above (d) None of the above

45. What are the Important Sites of Indus Valley Civilization? Choose the correct options –
I) Harappa in present Pakistan
II) Mohenjo–daro in present Afghanisthan
III) Ropar in Lahore
(a) Only I is correct (b) Both I & II are correct
(c) Both II & III are correct (d) All of the above

Passage(Q.46-Q.50): The central government has criticised India’s poor ranking on the Global Hunger Index
(GHI) 2022 and questioned the index’s methodology. The country has slipped further down the index over the
years and ranked an abysmal _____ out of _____ countries on GHI 2022. But how exactly is the GHI ranking
calculated? The Centre has been on a spree of rejecting methodologies of several global indices on democracy,
press freedom and now hunger for the second consecutive year. The government has accused the GHI of tainting
the country’s image and spreading misinformation — nearly duplicating its argument when the GHI 2021 was
released. India ranked _____ out of ______ countries last year and the Centre alleged “erroneous”
methodological issues were behind it. The premise of the Centre’s argument lies in the indicators being
inaccurate. It claims the report failed not only to capture ground reality but ignored the government’s efforts to
provide food security to the population through the public distribution system and other such efforts —
particularly so during the COVID-19 pandemic.

46. India has ranked _______ out of ________ countries in the Global Hunger Index (GHI) 2022.
(a) 95, 115 (b) 107, 121 (c) 102, 132 (d) 98, 110

Head Office: 127, Zone II, MP Nagar, Bhopal |+91-7676564400| https://www.toprankers.com Page 13 of 40
47. What is the Global Hunger Index? Choose the correct statements from the following –
I) The Global Hunger Index (GHI) is a tool for comprehensively measuring and tracking hunger at global,
regional, and national levels.
II) The GHI score is calculated on a 50-point scale reflecting the severity of hunger - zero is the best score
(implies no hunger) and 50 is the worst.
III) The GHI is prepared by European NGOs of Concern Worldwide and Welthungerhilfe.
(a) Only I is correct (b) Both II & III are correct
(c) Both I & II are correct (d) Both I & III are correct

48. GHI scores are based on the values of four component indicators. Choose the correct options –
I) Child Malnutrition
II) Child stunting
III) Child wasting
IV) Child mortality
(a) Both I & II (b) Both II & III (c) Both III & IV (d) II, III & IV

49. What is India’s performance in the Four Indicators? Choose the incorrect options from the following –
I) India’s child wasting rate (low weight for height), at 16.3%, is worse than the levels recorded in 2014 (15.1%)
and even 2000 (17.15%).
II) Prevalence of undernourishment has also risen in the country from 14.6% in 2018-2020 to 18.7% in 2019-
2022.
(a) Only I (b) Only II
(c) None of the above (d) All of the above

50. Under which Act It legally entitled up to 75% of the rural population and 50% of the urban population to receive
subsidised food grains under the Targeted Public Distribution System?
(a) National Food Security Act, 2013
(b) The Food Safety and Standards Act, 2011
(c) The Prevention of Food Adulteration Act, 1954
(d) Essential Commodities Act, 1955

Passage(Q.51-Q.55): The Times Higher Education (THE) World University Rankings 2023 has finally been
released and as expected, the [1] has once again topped the ranking list- for 7th time in a row. As per THE World
University Rankings, the [1] (UK) is followed by the University of Harvard (US) and the University of
Cambridge (UK).A total of 1,799 universities from as many as [2] countries and regions have been ranked this
year which is 137 more than last year's rankings. The US continues to dominate with 177 universities ranked
overall, including seven in the world's top ten, and 12 in the top 20. However, the number of US universities
represented in the top 100 continues to fall from 43 in 2018 to 34 this year.
Meanwhile, Asia is the most represented continent with 669 universities participating, jointly with Europe (639,
up 50 from 589 last year).

51. Which of the following will replace [1]?


(a) University of Oxford. (b) University College London.
(c) Stanford University. (d) Yale University.

52. Which of the following will replace [2]?


(a) 98 (b) 100 (c) 104 (d) 109

53. The ranking of the Universities are based on how many indicators?
(a) 3 (b) 4 (c) 5 (d) 6

Head Office: 127, Zone II, MP Nagar, Bhopal |+91-7676564400| https://www.toprankers.com Page 14 of 40
54. How many Indian institutions have been featured in Times Higher Education (THE) World University Rankings
2023?
(a) 68 (b) 70 (c) 75 (d) 81

55. Which among the following is India’s highest-ranked institute?


(a) Indian Institute of Science, Bangalore
(b) Indian Institute of science and technology, Mumbai
(c) Shoolini University of Biotechnology and Management
(d) Mahatma Gandhi University

Passage(Q.56-Q.60): India's existing power generation portfolio is the world's third-largest at nearly 360 GW,
and S&P Global Market Intelligence data shows another 203 GW of planned capacity. As part of India's long-
term decarbonization effort, Prime Minister Narendra Modi has pledged to add 500 GW of nonfossil generating
resources by 2030. Bloomberg NEF on June 22 estimated that India would need investments totaling $223 billion
to meet the 2030 target. S&P Global Ratings in an April report questioned whether the 2030 target is achievable.
Ratings noted that although India has been adding renewables at a rate of about 10 GW to 15 GW annually, the
country will not make a 2022 target of 175 GW. "The addition of new capacity will need to rise by at least double
for India to achieve 50% generation from renewables by 2030," Ratings said, adding that the country "lacks
comprehensive energy transition policies and a clear commitment to phase out of coal. "Ratings said India will
also need significant investment in expanding its transmission and distribution grid. In all, about $500 billion in
new spending will be required by 2030.

56. India is also targeting to reduce India’s total projected carbon emission by _____________by 2030, reduce the
carbon intensity of the nation’s economy by less than _____by the end of the decade, achieve net-zero carbon
emissions by 2070.
(a) 1 Million Tonnes, 34% (b) 1 Billion Tonnes, 45%
(c) 10 Million Tonnes, 50% (d) 50 Million Tonnes, 42%

57. How many solar parks of aggregate capacity of 37 GW have been approved in India?
(a) 45 (b) 30 (c) 55 (d) 62

58. What is the Present Status of Renewable Energy in India? Choose the correct options from the following –
I) The total installed capacity for renewable energy in India is 148.4 GW.
II) Solar Parks in Pavagada (2 GW), Kurnool (1 GW) and Bhadla-II (648 MW) are included in the top 5
operational solar parks of 7 GW capacity in the country.
III) The world’s largest renewable energy park of 30 GW capacity solar-wind hybrid project is under installation
in Assam.
(a) Only II is correct (b) Both I & II are correct
(c) Both II & III are correct (d) All of the above

59. What are the Challenges? Choose the correct options from the following –
I) Heavily Dependent on Exports
II) Raw Material Supply
(a) Only I is correct (b) Only II is correct
(c) None of the above (d) All of the above

Head Office: 127, Zone II, MP Nagar, Bhopal |+91-7676564400| https://www.toprankers.com Page 15 of 40
60. What are Government Initiatives? Choose the incorrect options from the following –
I) PLI scheme to Support Manufacturing
II) Domestic Content Requirement (DCR)
III) Imposition of Basic Customs Duty on import of solar PV cells & modules
IV) Modified Special Incentive Package Scheme (M-SIPS)
(a) Only I (b) Both II & III
(c) I, II & IV (d) None of the above

Passage(Q.61-Q.65): By resolution 49/214 of 23 December 1994, the United Nations General Assembly
decided that the International Day of the World's Indigenous Peoples shall be observed on 9 August every year.
The date marks the day of the first meeting, in [1], of the UN Working Group on Indigenous Populations of the
Sub-Commission on the Promotion and Protection of Human Rights. In 1990, the UN General Assembly
proclaimed 1993 the International Year of the World's Indigenous Peoples (A/RES/45/164, A/RES/47/75) Each
year, UNESCO marks the celebration of the Day by sharing information on projects and activities that are
relevant to the annual theme. Through partnerships with indigenous peoples, UNESCO seeks to support them in
addressing the multiple challenges they face, while acknowledging their significant role in sustaining the
diversity of the world’s cultural and biological landscape. UNESCO places the needs of indigenous peoples
amongst its priority areas for response.

61. The first meeting of the United Nations Working Group on Indigenous Populations was held in ____________in
________
(a) Washington, 1975 (b) Rome, 1990
(c) Geneva, 1982 (d) Paris, 1985

62. What is the theme of 2022? Choose the correct option -


I) “The Role of Indigenous Women in the Preservation and Transmission of Traditional Knowledge”
II) “Leaving no one behind: Indigenous peoples and the call for a new social contract”
(a) Only I (b) Only II (c) Either I or II (d) None of the above

63. Tribal people are concentrated in how many blocks in India? What are such areas designated as ?
(a) Tribal people are concentrated in 670 Blocks in India, Also the areas are designated as Unscheduled areas.
(b) Tribal people are concentrated in 720 Blocks in India, Also the areas are designated as protected areas.
(c) Tribal people are concentrated in 809 Blocks in India, Also the areas are designated as scheduled areas.
(d) Tribal people are concentrated in 825 Blocks in India, Also the areas are designated as prohibited areas.

64. What Steps has the Government of India taken for Tribal Welfare? Choose the incorrect options –
I) Digital Transformation of Tribal Schools
II) Development of Particularly Vulnerable Tribal Groups
III) TRIFED
IV) 10000 Springs Initiative
(a) Only I (b) II, III & IV
(c) Only IV (d) None of the above

65. What were the Major Recommendations of the Committee? Choose the correct options from the following –
I) The committee suggested launching a National Tribal Health Action Plan with a goal to bring the status of
health and healthcare at par with the respective State averages in the next 5 years.
II) The committee suggested nearly 80 measures to address the 12 priority health problems, the health care gap,
the human resource gap and the governance problems.
(a) Only I is correct (b) Only II is correct
(c) All of the above (d) None of the above

Head Office: 127, Zone II, MP Nagar, Bhopal |+91-7676564400| https://www.toprankers.com Page 16 of 40
SECTION - C: LEGAL REASONING

Directions (Q.66-Q.105): Read the comprehension and answer the questions:

Passage (Q.66-Q.70): Recently, a significant number of IT companies declared moonlighting as unfair and even
dismissed employees from their roles due to it. On the other hand, some other IT service companies introduced
a formal policy on moonlighting for its employees which allows them to earn financial benefits by undertaking
extrinsic projects. The act of undertaking work in more than one organisation, usually without the knowledge of
the employer, is known as moonlighting. Usually, a moonlighter engages in secondary employment due to fewer
amounts of wages in the existing job. As a result, moonlighters even accept underpaid jobs in their secondary
employment.

This implies that every moonlighting would take the form of two different contractual relationships.
Consequently, the permissibility of the second arrangement would depend upon the contractual terms of the
individual’s agreement with the engaging entities.
Indian law does not expressly define or deal with dual employment. On the other hand, prohibition with regard
to the inclusion of a non-compete clause is laid down in Section 27 of the Indian Contract Act, 1872. Such a
clause prevents an employee from starting their own business or accepting an offer from a competitor. It
eventually prevents the employee from competing with their employer, or leak data or confidential information
either during or after the period of employment.

In light of this, it can be stated that accepting dual employment is violative of the non-compete clause in an
employment contract. However, the position of Indian law on this issue is not completely settled. The Industrial
Employment (Standing Orders) Act, 1946 permits dual employment. Whereas under the Factories Act, 1948,
dual employment is prohibited. However, in some states, IT companies have been excused from this law.

66. "Mirage" is a fashion startup based in Gurugram that was founded in 2019, just before the corona virus pandemic
hit the country. The majority of its designers are freelancers who collaborate with the company to finalize the
designs. Meghna was one such student who worked with the company as an intern on a contract basis for three
months. Meghna's suggestions were considered, and all of the styles she recommended are now a big hit. Meghna
used the same method when she worked for another startup called "Chaprese." Mirage sued Meghna for breach
of contract, citing Section 27 of the Indian Contract Act, which prohibits employees from competing with or
working with their competitors. Decide
(a) As an employer, Meghna will be held liable because she cannot compete with her own employer.
(b) Meghna will not be held accountable because she was simply a student intern and not a company employee.
(c) Meghna will not be held accountable because she was only engaged on a contract basis, and was no longer
working for her previous company.
(d) Meghna will be held accountable since Section 27 forbids employee from engaging in any illegal competition
or unfair commercial practices.

67. Samaira works for an ed-tech company as a product head for one of its product lines called Creative agency. She
covers the product line's sales and marketing strategy from the ground up. One of her competitors contacted her
one day and asked her to work for them immediately at a higher salary. Samaira was aware of the company
regulation that she cannot join a competitor without giving a 50 days' notice before leaving the company or pay
an amount equal to two months' salary. Samaira accepted the job without informing her employer, and she
abruptly left the organisation. What will Samaira's liability be in this case?
(a) Samaira will be liable for moonlighting her employer for salary higher than that of his present salary.
(b) Samaira will not be held liable in absence of any breach of contractual violation.
(c) Samaira will be held liable as she abruptly left the organisation to work with a competitor, a clear violation
of section 27.
(d) Samaira will not be held liable in the present case as there was no data leak after the she left the former
employment.
Head Office: 127, Zone II, MP Nagar, Bhopal |+91-7676564400| https://www.toprankers.com Page 17 of 40
68. Continuing with the previous case, suppose Ayushi approached Samaira about giving her expertise in her ed-
tech startup, and Samaria agreed to offer her one hour daily after work in exchange for a 5 percent profit share
from Ayushi if the startup goes as envisioned. Determine Samaira’s liability in the current case?
(a) Samaira will be held accountable because she began attempting to develop a brand that will compete with
her existing organisation.
(b) Samaira will not be held responsible because she only gives her one hour that too after work.
(c) Samaira will not be held accountable because she is assisting Ayushi out of her own free choice and outside
the scope of her work.
(d) Samaira will be held accountable because it obviously violates her company's agreements.

69. It is the lockdown period, and Madhur's job as a finance adviser in an US based MNC only takes up 4-5 hours
of his day, but his firm does not allow him to do other jobs because it is against their company's corporate policy.
Regardless of his company's policy, Madhur also works as an accountant at his brother's startup venture because
his current employment barely meets his daily necessities in a metropolis like Bangalore. Can Madhur's employer
charge him for moonlighting?
(a) No, as will not be considered as moonlighting as he has enough time left on his plate which can be used for
some other alternate jobs to meet his daily needs.
(b) Yes, as Madhur cannot take other jobs even if he is not able to meet his daily necessity from his current
salary.
(c) No, as Madhur’s working as an accountant with his brother do not amounts to moonlighting.
(d) Yes, as Madhur’s working as an accountant with his brother’s firm amounts to moonlighting.

70. Assume that in the preceding situation, Madhur just served as an advisor in his brother's firm, for which he
received no additional compensation without intimidating this to his manager. In his defence, he claims that he
was simply assisting his brother and that this does not constitute moonlighting. Can the employer now bring a
case against Madhur of moonlighting?
(a) No, as he was no taking any additional remuneration from the second job.
(b) Yes, even if he was not charging anything from his brother’s firm he was engaged in a second employment.
(c) Yes, as his firm does not allow him to do other jobs because it is against their company's corporate policy.
(d) No, it would have been a case in which Madhur earned extra money by working at her brother's firm and not
otherwise.

Passage (Q.71-Q.75): Most employers in India still provide medical insurance only to the heterosexual spouse
of an employee and their children, and live-in or same-sex partners are not covered. This seems to be anomalous
with the changing legal landscape, and diversity, equity and inclusion policies becoming the norm worldwide.
Recently, the Supreme Court of India, in Deepika Singh versus Central Administrative Tribunal & Ors. (2022),
observed that though both legally and socially, the family is an unchanging unit consisting of a mother and father,
this needs to change with changing times, as many families do not fit this straitjacket. The court said that family
relationships might take the form of unmarried domestic partnerships or queer relationships, which should be
considered.

Admittedly, this was just an observation made by the court while deciding a case on maternity benefits, and thus
does not change the legal obligations of employers. But read with the court’s judgments on the rights of
transgender persons, the decriminalisation of same-sex relationships and the recognition of the rights of live-in
partners, it clearly shows the direction in which the law is evolving in India.
A quick look at the legal landscape of the United States, the United Kingdom and Europe shows that in all these
jurisdictions, the definition of family for the purpose of employee benefits has been widened, and the benefits
available to the heterosexual spouse of an employee are mostly also available to a same-sex spouse or a live-in
partner. Anything less may be considered discrimination and thus declared illegal.

Head Office: 127, Zone II, MP Nagar, Bhopal |+91-7676564400| https://www.toprankers.com Page 18 of 40
Even if the legal mandate in these jurisdictions does not prescribe inclusionary practices, the employers, under
their DE&I policies, are invariably accepting practices which try to make the workplace more inclusive by not
questioning the personal choices of employees regarding family, and extending the benefits like medical
insurance and parental benefits in a non-discriminatory manner.

71. Choose an appropriate statement according to your understanding of the passage, which determines the main
idea discussed in the passage by the author.
(a) There is no statutory requirement for accepting a broader meaning of family and extending the benefits, the
law also does not restrict employers from taking this approach.
(b) Recent court rulings show that the legal landscape is quickly changing, with several grey areas evolving.
(c) Not providing benefits to employees with personal choices that do not conform to traditional notions of
society takes away the sense of belonging from employees, leading to unhappy employees and higher
attrition rates.
(d) The recent judgments of the court can be considered and it can be concluded that not only mother and father
but family may include transgender couple, live in partners, etc

72. Malti and Meena are members of the queer community who wish to marry but are unable to do so because same-
sex marriage is still illegal in their country (lets assume india). Consequently, they live in a live-in relationship.
Let's say they decide to get a joint insurance policy. Will there application be acceptable?
(a) No, as there is no law on the matter defining a family to include same sex couples.
(b) Given the judges' prior comments in the courtroom regarding the inclusion of transgender couples, live-in
partners, etc. in the definition of family, the answer is yes.
(c) The answer is yes, as the judge made it clear that single people and couples living together but not married
should be taken into account when defining a family.
(d) No, but only if unmarried domestic partnerships or homosexual relationships are accepted as legitimate types
of family relationships.

73. Nandini lives in the US and works for a US company. Her spouse, Chandini, works remotely from the UK for
TOCO Pvt Limited, an Indian business. Chandini's medical insurance is paid for by Nandini's company as per
legal obligation of the employers. Suppose Chandini relocated for a few months to India and discovered that
her corporation only provides medical coverage to heterosexual spouses of its employees. What could possibly
be the basis for such a practice of discrimination?
(a) Employers are required by law to offer insurance coverage to heterosexual couples.
(b) Employers are required by law to offer insurance coverage to only married heterosexual couples.
(c) The legal systems in India and the US have quite different conceptions of what constitutes a family for the
purposes of employee benefits.
(d) In view of the current legal system, the corporation had continued to impose restrictions on itself even after
evolving rulings regarding the rights of live-in partners and transgender.

74. Jacky an Indian citizen works for an UK based company named Jerry and Co, which provides online counselling
to its customers mostly student, globally to understand the admission mechanism to different tier one universities.
Suppose Jacky was denied insurance benefit to his spouse citing that the Indian law does not have recognition
for the same. Is the contention valid?
(a) Yes, it is valid as India provides medical insurance only to the heterosexual spouse of an employee and their
children, and live-in or same-sex partners are not covered.
(b) No, as this seems to be anomalous with the changing legal landscape, and diversity, equity and inclusion
policies becoming the norm worldwide.
(c) Yes, as the company policies are directly related to the governing law of the nation.
(d) No, as in UK the definition of family for the purpose of employee benefits extend to a same-sex spouse or a
live-in partner.

Head Office: 127, Zone II, MP Nagar, Bhopal |+91-7676564400| https://www.toprankers.com Page 19 of 40
75. Imagine Jerry working for a US company that does not mandate inclusionary policies that extend the
employer benefits offered to a heterosexual employee's heterosexual spouse to a same-sex spouse or a live-in
partner. But his company does have a diversity, equality, and inclusion (DE&I) policy, and it serves three
purposes: Promote an inclusive, diverse, and harassment-free and discrimination-free workplace. Jerry, who is
gay, experienced prejudice when his supervisor forbade him from going on a business trip because Jerry is a
member of the LGBT community. What legal advice would you offer Jerry in this situation?
(a) Jerry has the right to speak out about his supervisor's actions on the grounds of discrimination.
(b) Jerry is only permitted to complaint about violation of DE&I policy violations.
(c) Jerry will not be able to pursue any legal recourse in this situation.
(d) Jerry won't have any recourse in the current situation because his country has no laws governing this specific
issue.

Passage (Q.76-Q.80): The Hindu Law of Maintenance has been codified under the Hindu Adoption and
Maintenance Act of 1956. According to Sec. 3 of the Act maintenance includes: In all cases provision for food,
clothing, residence, education and medical attendance and treatment.
In the case of unmarried daughter, also the reasonable expenses, of her marriage, and expenses incidental to her
Marriage expenses cover the actual expenses in performing the marriage and expenses incurred in the betrothal
function and nuptial ceremonies. The obligation of a Hindu to maintain others arises on account of personal
relationship or ownership of property. Personal obligation is recognized in favour of virtuous wife, infirm, and
aged parents, and minor children. The personal obligation is a legal obligation, existing irrespective of ownership
of property. Obligation based on ownership of property relates to other members in the Joint Hindu Family, like
the females.

The section enacts that the wife loses her right to maintenance, if she becomes unchaste or becomes a convert to
another religion. However, under Old Law bare pittance or starving maintenance was allowed to an unchaste
wife or separate maintenance is altogether denied. But the husband's condonation of the wife's unchastity can
cure her disqualification. In the case of conversion to another religion, the wife cannot recover her right to
maintenance, by reconversion to Hinduism.

Maintenance of illegitimate children-Under old Law an illegitimate son could claim maintenance from his
putative father. After the act he can claim it also from his mother. When his father died, he could receive
maintenance of his life time out of his father's property. This right was recognized as substitute for a share. Under
the Old Law illegitimate son by non-Hindu concubine could not claim maintenance. Now also Sec. 24 requires
that a claim to maintenance should be a Hindu, and thus there is no change in the law.

76. Mohini and Mahant both work for an MNC in Bangalore. They have different sets of office buddies because
they work in the same office but for different team leaders. Mahant was forced to accept the reality that his wife
was forging friendships with other male colleagues despite not liking it. There was a rumour he heard in the
office that goes that Mohini might be dating Mohan. Mahant overlooked the fact and didn’t say a word to
Mohini but he began to stop paying his half of her maintenance expenses. Invoking Section 3 of the 1956 Hindu
Adoption and Maintenance Act, Mohini brought a lawsuit. How well will she do?
(a) She will win in this case since she did nothing that would prevent Mahant from paying the maintenance to
which she is legally entitled.
(b) She will lose since she will no longer be eligible to get maintenance from Mahant if she has an extramarital
affair with Mohan.
(c) She would win as a Personal obligation is recognised in favour of virtuous wife.
(d) She would lose because Mahant stopped covering Mohini's expenses once Mohini stopped being faithful.

Head Office: 127, Zone II, MP Nagar, Bhopal |+91-7676564400| https://www.toprankers.com Page 20 of 40
77. Assuming the identical facts as above, let's say Mahant filed for divorce on the basis of the rumor that she might
be having an affair with his colleague Mohan and stopped paying Mohini any maintenance while the case was
pending. Is Mohini entitled to maintenance in this situation alleging Maintenance in favour of a virtuous wife?
(a) Yes, as the allegations of chastity had not yet been proven and she was still the Manhant’s wife.
(b) No, as the wife loses her right to maintenance, once she becomes unchaste.
(c) Yes, given that it was only a rumor and Mohini is still Manhant's wife, there are no questions. Personal
responsibility is acknowledged in the wife's favour.
(d) No, Mohini cannot request maintenance while claiming to be a virtuous wife until the accusations made
against her are proven false.

78. Let's say the rumour that Mohini and Mohan were having an extramarital affair was true, and Mohini secretly
converted to Mohan's religion, X, and began performing all the rituals without notifying Mahant. After 16 years,
Mahant learned that Mohini was having an extramarital affair with his coworker Mohan. However, they came
to an agreement whereby Mahant agreed to pay Mohini regular maintenance in exchange for her staying with
him because of Manhant’s reputation in the community and his desire to keep the affair a secret from his family.
Is this arrangement valid with respect to maintenance received by Mohini?
(a) No, it is not valid as Mohini converted herself to another religion X.
(b) Yes, as husband's condonation of the wife's unchastity can cure her disqualification from claiming the
maintenance.
(c) No, as the wife loses her right to maintenance, once she becomes unchaste.
(d) No, as the wife cannot claim right to maintenance once she converts to another religion.

79. Suppose, Mohini and Mohan had a child named Mickey. Mickey stays with Mohan. Both Mohini and Mohan
was not married for some personal reasons at the time of Mickey’s birth. Mickey will be the responsibility of?
(a) Mickey can only claim maintenance from her father only as per the old law.
(b) Mickey can claim maintenance from her father and mother both as per the new law.
(c) Mickey can claim maintenance from his mother too as per the new law.
(d) Mickey responsibility will be borne by both of his parents as per the law.

80. Keeping the facts within the bounds of old law, it was considered customary for a Hindu to engage in bigamy.
Mahant held a concubine named Sujatha in addition to his one wife Sonam. He had a son named Darry from
Sujatha. Mahant has rejected Darry's request for maintenance on the grounds that only legitimate sons are eligible
for it. Is it correct?
(a) No, as both legitimate as well as illegitimate son can claim maintenance from their father under the old law.
(b) Yes as only a legitimate son can claim and is entitled to maintenance.
(c) No, as under the old law an illegitimate son can claim maintenance from his putative father.
(d) Yes, but under the old law an illegitimate son can also claim maintenance from his putative father.

Passage (Q.81-Q.85): Cheating is defined under Section 415 of the Indian Penal Code as whoever fraudulently
or dishonestly deceives a person in order to induce that person to deliver a property to any person or to consent
to retain any property. If a person intentionally induces a person to do or omit to do any act which he would not
have done if he was not deceived to do so and the act has caused harm to that person in body, mind, reputation
or property, then the person who fraudulently, dishonestly or intentionally induced the other person is said to
cheat. Any dishonest concealment of facts which can deceive a person to do an act which he would not have
done otherwise is also cheating within the meaning of this section.

One of the important ingredients which constitute the offence of cheating is deception. Deception can be done
to induce the other person to either deliver or retain the property or to commit an act or omission. Deceiving
means to make a person believe what is false to be true or to make a person disbelieve what is true to be false by
using words or by conduct.

Head Office: 127, Zone II, MP Nagar, Bhopal |+91-7676564400| https://www.toprankers.com Page 21 of 40
In deception, a fraudulent representation or wilful misrepresentation of a fact is made directly or indirectly with
an intent to commit the offence of cheating. In order to prove the offence, it is not only important to prove that
a false representation was made by the accused but also that the accused had the knowledge that the
representation was false and wilfully made it in order to deceive the prosecutor.

81. Chaitanya and Chirag were best friends since childhood. While Chaitanya was born and brought up in a city,
Chirag was born and brought up in Chaitanya’s native village. Chaitanya one day urged Chirag to post a picture
on his profile on Finstagram by incorrectly telling him that the post is only visible to Chaitanya. On finding out
that his post is visible to all users of Finstagram, Chirag sues Chaitanya under S.415 of the IPC. Decide the
validity of the suit.
(a) Chaitanya would be held liable under S.415 of the IPC as he deceived Chirag into posting his photo knowing
very well that had he not induced Chirag to post the photo he wouldn’t have done so.
(b) Chaitanya would not be held liable under S.415 as his deception in no way affected Chirag’s mind, body,
reputation or property as required by the law.
(c) Chaitanya would not be held liable as the ingredient of deception is absent in his acts.
(d) Chaitanya would be held liable as he wilfully deceived Chirag about the viewership of the photo in spite of
knowing the truth himself and also being cognizant that it was only because of his statements that Chirag
posted a picture.

82. Belur and Co. was a Chandigarh based travel agency that specialised in solo travelling to the Himalayas known
for their extreme weather conditions and wildlife. Ishika, a student asked them to book a trip to the Himalayas
but ensure that she doesn’t visit any area that had any sightings of wildlife. The agency booked a trip for her but
hid the fact that the trek she was going for recently had been the location of the first ever yak migration. After
her trip, Ishika was made aware of the fact and now wants to sue Belur and Co. Advise her.
(a) Ishika should not file a suit as she would fail to prove cheating under S.415 of the IPC as there is no wilful
representation on part of the travel agency.
(b) Ishika should file a suit as her plea would be successful considering there was an active concealment of facts
on part of the agency and such concealment had affected her decision of going on the trek.
(c) Ishika should not file a suit as her plea would be unsuccessful as the actions of the agency did not cause
Ishika to retain or part with any property, had it not been for the agency’s representation.
(d) Ishika should file a suit as she would be successful in proving the ingredients of deception and fraudulent
representation as required under S.415 and S.420 of the IPC.

83. Priya and Siya were the landlords of a house that Niya was wanting to rent out. While Niya liked the house, she
questioned the landlords whether there is a pigeon nest anywhere close to the house as she is allergic to birds.
Priya and Siya denied the same since they didn’t have any knowledge about the issue. Niya signed the agreement
in 2019 and was diagnosed with acute breathing issues in 2022 that were triggered due to proximity to birds. She
filed a case against the landlords under S.415 of the IPC. Decide.
(a) The case would be decided in Niya’s favour as an essential fact was concealed from her which would have
altered her decision.
(b) The case would be decided in the landlord’s favour as they lacked the intention to deceive Niya and cause
her any harm.
(c) The case would be decided in Siya and Piya’s favour as they lacked the knowledge that their representation
of pigeon nests being absent was incorrect and thus, their act lacked the ingredient of wilful
misrepresentation.
(d) The case would be decided in Niya’s favour as there was a wilful misrepresentation on part of the landlord’s
as they wrongly informed Niya in spite of not knowing about the information’s veracity.

Head Office: 127, Zone II, MP Nagar, Bhopal |+91-7676564400| https://www.toprankers.com Page 22 of 40
84. Srishti and Yash were entering into a joint venture revolving around automobiles. While Srishti was an MBA
graduate, Yash was an automobile engineer. Pranjal an investor in the automobile market was willing to invest
in their business if they could supply 500 models of X car in 1 month. Srishti on asking for Yash’s advice was
told by Yash that the assignment was doable in spite of himself also knowing that it was impossible. Srishti lost
money due to taking up the project and now wants to sue for cheating. Decide.
(a) Srishti can sue Yash and Pranjal both as they wilfully misrepresented the possibility of the assignment being
completed in due time.
(b) Srishti can only sue Yash as it was his wilful misrepresentation on basis of which she acted in the way she
did.
(c) Srishti can sue Pranjal as she was aware that her demand could not be satisfied and deceived Srishti into
falsely believing that it could.
(d) Srishti would not be able to sue Yash as it is nowhere indicated that she was acting solely on Yash’s advice
and without his representation would not have taken the actions she did.

85. Which of the following statements if true, would cast the least serious doubt on the scope of S.415 of the IPC?
(a) Harm to mind, body, reputation or property is a necessary ingredient of cheating.
(b) Any representation made on basis of unknown facts would be considered to be a false representation.
(c) Wilful misrepresentation is immaterial as long as there was an intention to deceive.
(d) It is not necessary that the victim of cheating was solely or only acting on the deception of the accused.

Passage (Q.86-Q.90): When due to the negligence of the Tortfeasor, any dangerous thing escapes from his/ her
premises and harms anyone/ anything( including neighbor), then the liability is said to be strict liability.
The essential ingredients under strict liability are as follows:

Dangerous Thing: The strict liability rule applies to 'Anything that can do mischief if it escapes.' The essential
feature that serves as the basis of applicability is that the word 'anything' refers to substances accumulated by
the defendant and brought by him to his property and not naturally occurring substances. The Courts usually
use a fact-based test in determining the 'dangerous thing' to form an analysis as to whether the thing is likely to
cause danger or mischief if it escaped into the land's surroundings. Strict liability has three categories; animals
(owned or possessed), abnormally dangerous activities, and product liability. The plaintiff has the burden of
proof and have to prove that the damage and harm were foreseeable by the defendant.

Unnatural Use of Land: The strict liability rule will apply if the defendant collects and operates any substance
likely to cause mischief if it escapes. The storage of large quantities of dangerous materials, the casual way of
its maintenance, and the character of the neighbourhood are characters that go into circumstantial evidence
depending on which liability may be owed.

Escape: The mere evidence of a 'dangerous thing' is not enough to prove the defendant is liable, that substance
must escape from the premises of the defendant to another's and inflict harm to the victim. The word 'escape'
denotes to signify an escape from the place of the defendant or had control or owe it to a place which is outside
his control or occupation.

86. Himanshu and Madhav were known to have the best organic honey, which they used to harvest from their own
bee hive farm. They hired a professional to take care of the bee hive farm that was located around 30 km from
the city of Nagpur, in the outskirts. The professional informed Himanshu and Madhav that the hives should be
covered during the monsoons since not doing so will clearly cause the bees to grow violent and abandon their
hives. The owners didn’t pay heed to the same and in the middle of the monsoon, the bees from their farm stung
school children at a nearby school. Decide their liability.
(a) The professional would be held liable as he was clearly aware of the harm and damage that could follow the
act of not covering the hives and still did not insist on taking actions that could prevent the same.

Head Office: 127, Zone II, MP Nagar, Bhopal |+91-7676564400| https://www.toprankers.com Page 23 of 40
(b) There would be no liability in the present case since the ingredient of unnatural use of land is not being met
since the hives were maintained by a professional and the nature of the neighbourhood was one of the
outskirts, which implies that there was no foreseeable harm to large group of people or property.
(c) The owners would be liable as the bees, dangerous things under owned animals in this case, escaped from
their farm and caused harm to the school students due to them wilfully not listening to the professional’s
advice.
(d) The owners and the professional in-charge of the hives would both be liable under strict liability, the former
because of their negligence and the latter for not insisting enough on taking necessary precautions.

87. Ghaziapur was a small town located on the Western coast of India and was known for its maritime climate and
incidents of natural disasters, primarily earthquakes. Akshat was resident of Ghaziapur and owned a brick kiln.
He used to store large amounts of sand at any point in the earthquake proof storage (required timely maintenance)
in the compound of his kiln. Under which of the circumstances given below would Akshat not be held strictly
liable for any harm arising out of the escape of the sand.
(a) Akshat’s brick kiln is located in a recently notified residential area of Ghaziapur as a result of which the
escape of the sand has caused several breathing issues to the residents living near the brick kiln.
(b) A major earthquake hit the town of Ghaziapur leading to the collapse of several structures especially the
ones that were weak. Akshat’s storage was maintained regularly and was one of the structures that collapsed
in the industrial area.
(c) Akshat had delayed the repairs of the storage unit for the sand after multiple reminders from his personal
assistant. A very minor earthquake in 2021 caused serious cracks to occur in the unit and in 2022 the unit
collapsed.
(d) The storage unit was in excellent condition and had a storage capacity of nearly 100 tonnes but when the
escape occurred was filled up to 150 tonnes, for a short duration till a consignment was delivered.

88. Vedika and Anupama were owners of neighbouring cattle farms and the boundary between their farms was a
short fence, an arrangement that was there for the last 10 years. One day two cows from Anupama’s farm entered
into Vedika’s farm and grazed on a patch of wheat. Vedika wants the civil courts to hold Anupama strictly liable.
Advise her on whether she should approach the courts or not.
(a) Vedika should approach the courts as there has been negligent escape of dangerous things, in this case cows
owned by Anupama and there is harm in form of destroyed wheat crops.
(b) Vedika should not approach the court as the burden of proving harm and negligence would fall upon her
which in this present case is nearly impossible to prove.
(c) The ingredient of unnatural use of land would not be met as the existence of cows in a farm neighbourhood
is not considered unnatural. Moreover, Vedika should not approach the court as there is no explicit
negligence on part of Anupama the given fact scenario discloses.
(d) Vedika should approach the court as the use of land is unnatural since the cows were not maintained as they
were clearly not given enough to graze, forcing them to enter Vedika’s farm and escaping Anupama’s farm.

89. Suryansh was a tight rope walker who used to often perform at the Cirque Du Soleil. He would often practice
his art in the compound of his housing society and set up his equipment in the evenings. One such evening, when
he was practicing, the tight rope snapped and the lose end hit another resident, Manya in the face. Decide the
liability of Suryansh.
(a) Suryansh would be held strictly liable provided Manya can prove that there was negligence on his part in
maintaining his equipment which in turn caused the rope to snap and hit Manya in the face.
(b) Suryansh can be held strictly liable as there is unnatural use of land of a housing society by practicing
tightrope walking therein, along with an escape by the snapping of the rope, a product liability.
(c) Suryansh cannot be held strictly liable as he could not possibly foresee the harm that could ensue due to the
snapping of his tightrope and the foreseeability on part of the tortfeasor is an ingredient to show strict
liability.

Head Office: 127, Zone II, MP Nagar, Bhopal |+91-7676564400| https://www.toprankers.com Page 24 of 40
(d) Suryansh would be held liable as there has been an escape of the tightrope from his control which also
qualifies as escape under the passage.

90. Which of the following statements if true, would be the most plausible reason to introduce the concept of strict
liability in tort law?
(a) The concept of strict liability was introduced to ensure that the duty of care one owes to their neighbour is
abided with and any such breach that causes damage is duly addressed.
(b) Strict liability was introduced with the aim of penalizing any sort of harm that even did not arose out of the
actions of a person, negligence being immaterial.
(c) Prior to the introduction of strict liability, any escape of goods outside the premise of a person and the
subsequent harm it caused were covered by negligence which often didn’t allow for sufficient compensation.
(d) This concept was introduced into the Indian civil law in order to ensure big enterprises, entities that are
usually involved in dangerous goods and processes, are held liable for any fault of theirs without leaving any
loophole for them to exploit.

Passage (Q.91-Q.95): In a democratic society, even the rights of the accused are sacrosanct, though accused of
an offence, he does not become a non-person. Rights of the accused include the rights at the time of arrest, at the
time of search and seizure, during the process of trial and the like. The constitution of India guarantees every
person right against self incrimination under Article 20(3) stating that no person accused of any offense shall be
compelled to be a witness against himself. It is well established that no one can forcibly extract statements from
the accused, who has the right to keep silent during the course of interrogation. According to Section 50(1) of
CrPC, every police officer, arresting any person without warrant shall forthwith communicate to him full
particulars of the offence for which he is arrested or other grounds for such arrest. Secondly, when a subordinate
officer is deputed by a senior police officer to arrest a person under Section 55 of CrPC, such subordinate officer
shall, before making the arrest, shall notify the person to be arrested, the substance of the written order given by
the senior police officer specifying the offence or other cause for which the arrest is to be made. Non-compliance
with this provision will render the arrest illegal. Thirdly, in case of arrest to be made under a warrant, Section 75
of CrPC provides that the police officer executing a warrant of arrest shall notify the substance thereof to the
person to be arrested, and if so required, shall show him the warrant. If the substance of the warrant is not notified,
the arrest would be unlawful. Section 50A makes it obligatory on the part of the police officer not only to inform
the friend or relative of the arrested person about his arrest but also to make entry in a register maintained by the
police. Hence, in light of the discussed provisions, a police officer must make sure that handcuffs are not used
unnecessarily, the accused is not harassed needlessly, the arrested person is made aware of the grounds of his
arrest, informed whether he is entitled to bail and of course produced before a magistrate within twenty-four
hours of his arrest.

91. H was accused of committing an offense and was urged to comply with the police officers. Even after multiple
requests, he was not informed of the grounds of his detention and was not presented with any arrest warrants
either. As a result, he refused to cooperate with the authorities, but he was coerced into going to the police station,
where they asserted that the reasons for his arrest were not being disclosed in the public interest. As a
consequence, he filed a complaint alleging violation of his rights under Section 50 of CrPC. What are your
thoughts on the entire factual scenario?
(a) H cannot assert any abuse of his rights since it was against public interest, which is an exception granted in
Section 50 of the CrPC.
(b) Since he wasn't injured or interfered with in any way while the police officers were making the arrest, the
arrest will be deemed legitimate.
(c) Since he was not communicated of the grounds for his detention, the arrest will not be regarded as
being consistent with Section 50.
(d) H may argue that his rights were infringed as a result of being forced against his will to cooperate with the
police.

Head Office: 127, Zone II, MP Nagar, Bhopal |+91-7676564400| https://www.toprankers.com Page 25 of 40
92. M was charged with the offense of murder. Y visited him at home, presented an arrest warrant, and briefed M of
the grounds for the arrest. He also presented the evidence that backed it up. M declined to answer any questions
when he was taken in for interrogation. As a result, the police officers contended that his silence would be
construed as a speech if he refused to cooperate with them and remained silent. Would you assert any rights’
violations if you were the defendant in this case?
(a) He was adequately informed of the grounds for his detention and was presented the necessary arrest warrant,
thus there is no basis for a claim of breach of rights.
(b) M may contend that his rights were infringed as a result of the fact that his silence was not regarded with the
appropriate consideration and was interpreted as a confession.
(c) Since the accused cannot be allowed to keep silent in cases of heinous crimes such as murder, there is no
basis for a contention that the rights of the accused were violated.
(d) M can argue that since he was made to appear to the police station for questioning, his rights have been
compromised.

93. Which of the following situations constitutes not a violation of the rights of an accused?
(a) P was detained at the police station for 36 hours without being interrogated despite being accused of the
crime of culpable homicide amounting to murder.
(b) P was arrested for the crime of extortion, produced with the necessary arrest warrant, and compelled to
confess to the crime.
(c) P was taken to the police station for interrogation after being accused of the offense of criminal
misappropriation.
(d) P was detained without a warrant and was not made aware of the specifics of his detention after being found
guilty of abducting the magistrate's daughter.

94. When certain evidence against T came to light, G's colleague and friend, P asked him to go and detain T since it
was imperative that he be held in custody otherwise he would continue committing the same offense repeatedly.
Following P's advice, G went to T and made an arrest without showing a warrant for the arrest or stating the
grounds for the arrest. T filed a complaint after learning about his rights, alleging that the requirements of Section
55 of CrPC were not adhered to. Analyze.
(a) Since Section 55 mandates that the accused be informed of the grounds for his arrest, T's rights are being
disregarded.
(b) Considering that the prerequisites outlined in Section 55 are not fulfilled, T's rights are not being infringed.
(c) T should have objected about the breach prior to his arrest rather than later, in accordance with the act's
requirements, therefore T's rights are not being violated.
(d) Since G did not present an arrest warrant when conducting the arrest, T's rights have been violated.

95. Which of the following statement is in line with your understanding of the given passage?
(a) It is widely established that one has the right to coercively extract testimony from an alleged offender who
has the right to remain silent during interrogation.
(b) Every police officer must under Section 50, immediately inform someone they arrest with a warrant of the
entire details of the crime they are suspected of committing or other reasons of why they are being detained.
(c) According to section 55, the subordinate officer has a duty to tell the person who has been arrested of the
details of the written order from the senior police officer stating the crime or other basis for the arrest.
(d) As per Section 50A, the detaining party's friend or relative cannot be informed of their arrest, and the police
officer must also record the arrest in a register that is kept by the police.

Head Office: 127, Zone II, MP Nagar, Bhopal |+91-7676564400| https://www.toprankers.com Page 26 of 40
Passage (Q.96-Q.100): Right to Education has been a part of the Directive Principles of State Policy as part of
Chapter 4 of the Indian Constitution. This right was made enforceable by putting it in Chapter 3 of the
Constitution as Article 21. This entitles children to have the right to education enforced as a fundamental right.
This Article lays down states’ goal of providing free and compulsory universal education to all the children until
the age of 14. Every child in the age group of 6-14 years will be provided 8 years of elementary education in an
age appropriate classroom in the vicinity of his or her neighbourhood. Any cost that prevents a child from
accessing school will be borne by the state which shall have the responsibility of enrolling the child as well as
ensuring attendance and completion of 8 years of schooling. No child shall be denied admission for want of
documents; no child shall be turned away if the admission cycle in the school is over and no child shall be asked
to take an admission test. Children with disabilities will also be educated in the mainstream school. The Indian
judicial system attempted to include the right to education as a component of the Right to Life in the 1993 case
of Unni Krishnan v. State of Andhra Pradesh. Through a constitutional amendment that was passed in 2002, the
Indian Parliament also gave its future inhabitants the right to an education. To give effect to Article 21A of the
Constitution, the Right of Children to Free and Compulsory Education Act, 2009, was passed. It said that the
state would provide free and mandatory education to children between the ages of 6 and 14 years old,
incorporating the right to primary education. In 2008, six years after the Indian Constitution underwent an
amendment (86th Amendment, 2002); the Cabinet approved the Right to Education Bill.

96. X was the lone breadwinner in a household of six people. He had three siblings as well as physically handicapped
parents. He discontinued his degree since he wanted to help his siblings with their education. He had managed
to finish class eleven and was unable to graduate because of work obligations. His father advised him to seek for
the free education provided by Article 21A given that he was an excellent student in school and genuinely
interested in learning. How should he exercise his rights, in your opinion?
(a) X should get access to free education as provided under Article 21A since every child has the right to
education enforced as a fundamental right.
(b) X should not get access to free education since the requisites under Article 21A are not being adhered to.
(c) X should get access to free education since the state shall have the responsibility of enrolling the child as
well as ensuring attendance and completion of eight years of schooling.
(d) X should not get access to free education since he discontinued the degree at his own will and can no longer
claim it under Article 21A.

97. P was from a financially and educationally underprivileged family. Only two meals a day were managed by him.
Due to his family's hardship, he made the decision to enrol himself in school while also working part-time after
acquiring knowledge for a few years. He made the decision to apply for the Article 21A scheme since he lacked
the funds for education. In order to graduate from the University of California, R, the education minister's son,
applied under the same scheme. Is R qualified to do so? Give a convincing explanation to support your argument.
(a) R will be entitled for the free education scheme since everyone in the state shall receive free and compulsory
education that includes the right to primary education.
(b) R will not be considered for the free education initiative since he does not satisfy the prerequisites to be
eligible for it.
(c) R will be permitted to use the scheme since children's right to an education is recognized as a fundamental
right.
(d) R is not entitled to assert his article 21A entitlement to free education because only citizens that attain
education in India are admissible for the benefit.

Head Office: 127, Zone II, MP Nagar, Bhopal |+91-7676564400| https://www.toprankers.com Page 27 of 40
98. H had just turned twelve. Since the prior school's schedule did not work for him, he made the decision to switch
schools. He subsequently submitted the application forms and was asked to give admission test for multiple
private schools. Some of them shortlisted him, while the others rejected him. His application was turned down
by the school that was his preferred choice. So, bearing in mind the clause from Article 21A that no child shall
be required to take an admission test, he filed a complaint against the institution. Analyze.
(a) H is correct to assert that Article 21A guarantees him rights because it states clearly that no child shall be
required to take an admission test for private school.
(b) The benefit outlined in Article 21A shall not be provided to H because private schools are not covered by
the provision.
(c) H is not entitled to exercise his right under Article 21A because it is unclear from the passage whether he
qualifies as a beneficiary or not.
(d) Since all of the conditions of Article 21A are being fulfilled, H shall pursue his entitlement under the
aforementioned legislation.

99. K was very disinterested in academics. He wanted to excel in sports and therefore decided to quit school. His
parents tried to convince him a lot but in vain. K, as a result, got himself admitted into the town’s top sports
institute. His parents concerned for his future, complained about him under Article 21A claiming that it is the
states’ obligation of providing free and compulsory universal education to all the children until the age of 14.
Share your opinion about the case in hand referring to the aforementioned passage.
(a) Since the state is required by law to provide free education to all qualified individuals, K's parents may sue
the state for damages.
(b) Considering that only the affected student may use and claim the benefit, K's parents cannot hold the state
accountable.
(c) Due to the fact that the instance in question satisfies all of Article 21A's requirements, K's parents may
petition the state.
(d) Since the state guarantees free education to children between the age of 6-14 year under Article 21A and
state did not deny K the benefit of the same, K's parents cannot hold the state liable.

100. Select the option that, in your estimation, accommodates the information given in the passage.
(a) Any cost that precludes a child from accessing education will be covered by the government school, which
will be responsible for registering the child and making sure they attend and finish their eight years of
education.
(b) As stipulated in Chapter 3 of the Indian Constitution, the right to education is one of the Directive Principles
of State Policy. By incorporating it as Article 21 in Chapter 4 of the Constitution, this right became legally
binding.
(c) No child shall be refused entry due to a lack of documentation, nor shall any child be turned away if the
school's admission cycle has ended. Children with impairments will also receive access to education in a
regular setting.
(d) In the 1993 case of State of Andhra Pradesh v. Unni Krishnan, the Indian court system added the right to
education as a component of the Right to Life. The Indian Parliament also granted the right to the country's
future citizens by a constitutional amendment that was approved in 2002.

Head Office: 127, Zone II, MP Nagar, Bhopal |+91-7676564400| https://www.toprankers.com Page 28 of 40
Passage (Q.101-Q.105): Before we can proceed to evaluate the circumstances in which a defence can be used
in any tort case, it is absolutely essential to understand what the word ‘defence’ me(c)onventionally the word
defence is used to refer to those arguments which when used persuades the court to conclude that the defendant
in a case is not guilty. So, they basically include ‘absent element defences’ which are denials of the components
of the tort that the defendant has allegedly committed. Now we shall see two commonly known and recognized
defences to any tort. (a) Consent: When a tort is committed, meaning that a defendant’s actions interfered with
the plaintiff’s person or property, a plaintiff’s consent will excuse the defendant of the wrongdoing. Although a
defendant’s conduct may be considered immoral, or harmful, if the plaintiff allows these interferences to occur,
then the defendant is not considered to have committed a tort. Consent occurs when a plaintiff displays a
willingness to participate in the defendant’s conduct. This consent can be expressed or implied. The defendant
may infer consent from the plaintiff’s actions the way any reasonable man would. In some cases, silence and
inaction may manifest consent when it is reasonable to assume that a person would speak or act if he objected to
the defendant’s actions. Consent may not always excuse a defendant of liability. Sometimes consent is ineffective
under certain conditions. If the plaintiff lacks the capacity to consent, is coerced into consenting, or consents
under false pretences, the consent is not valid as a defence to the tort. (b) When the plaintiff is a wrongdoer: The
law excuses the defendant when the act done by the plaintiff was illegal or wrong. This defence arises from the
Latin maxim ‘ex turpicausa non oritur action’ which means no action arises from an immoral cause. So an
unlawful act of the plaintiff itself might lead to a valid defence in torts. This maxim applies not only to tort law
but also to contract, restitution, property and trusts.

101. Q requested that E lend her the Air fryer machine that she had. E wanted to get rid of it because the machine had
been dormant for a long time. The machine suffered some internal damage, which caused it to stop functioning.
E lent the machine to Q. Q received a severe electric shock when she turned it on and began her task. The
machine had also blasted and was in no condition to be fixed. As a result, E filed a lawsuit against Q for monetary
damages. What are your thoughts on the current situation?
(a) E is entitled to compensation since Q should have been more cautious when using the equipment.
(b) E cannot sue for damages because she gave the malfunctioning machine to Q without alerting her.
(c) E cannot sue for damages because the machine was already dysfunctional, therefore even if the machine was
blasted, it makes no difference.
(d) E is entitled to compensation because Q was responsible for determining whether the machine was fit to run.

102. Choose the option that you consider is the most irrational based on the information in the passage.
(a) Consent is demonstrated by a plaintiff's willingness to participate in the defendant's conduct. This approval
might be expressed or implied.
(b) When the plaintiff's action is illegal or wrong, the law excuses the defendant. As a result, an unlawful conduct
committed by the plaintiff may result in a viable defence in torts.
(c) If the plaintiff lacks consent capacity, is coerced into consenting, or consents under false pretences, the
consent is not invalid as a defence to the tort.
(d) A defendant's actions may be considered immoral or detrimental. If the plaintiff permits these interferences,
the defendant is not deemed to have committed a tort.

Head Office: 127, Zone II, MP Nagar, Bhopal |+91-7676564400| https://www.toprankers.com Page 29 of 40
103. H completed her graduation in another city. During the New Year's break, her friend P asked her to bring a box
of dark chocolate biscuits worth Rs.1200. P stated that she would pay as well. H looked around for the same dark
chocolate biscuits, but they were all out of stock. Instead, she purchased chocolate cookies at Rs.800 but told P
they were also worth Rs.1200. When she demanded payment, P declined, stating that the cookies she delivered
were not the same as the cookies P had asked her to bring. H argues that P agreed to bring the cookies and that
she now owes the money. Decide?
(a) P is compelled to pay the sum because she requested that H deliver a box of cookies.
(b) P is required to pay the sum because the difference between chocolate and dark chocolate biscuits is minimal.
(c) P is not bound to pay the sum because the consent for the chocolate cookies was not obtained prior to the
purchase.
(d) P is not obligated to pay Rs.1200, but she is required to pay Rs.800, the original price of the chocolate
cookies.

104. Y wanted to rent a property for a few weeks. After much research, he came across Z's property. Z had already
notified Y about the low hanging cables and the leakage in the bathroom during the tenancy contract. He also
paid him Rs.5000 to get it fixed before moving in. Y didn't bother getting it repaired because he just planned to
live for two weeks and spent most of his time at work. When Y woke up on a Sunday morning, he decided to
order his breakfast. He slid coming out of his room owing to the water leakage, and when he tried to balance, he
clutched the wires in place and received an electric shock. He filed a lawsuit against the landlord, demanding a
refund of the rent as well as reimbursement for his medical bills. Comment
(a) Y can sue for damages because it was the landlord's responsibility to fix the cables and leak in the dwelling.
(b) Y cannot sue for damages because he should not have picked that apartment in the first place.
(c) Y is entitled to compensation because he was not anticipating the water to spill on the floor.
(d) Y cannot claim damages and the landlord can claim that the plaintiff is the wrongdoer because he had already
paid him the repair charges.

105. In which of the following situations can the defendant use the plaintiff's consent as a defence?
(a) F went to T's house when she was sleeping and asked if she may borrow her dress. T saw F's silence as
consent because she did not object.
(b) Y intended to borrow an economics book from Z and asked if he might borrow the chemistry book; when Z
objected later, Y claimed consent.
(c) K asked W if he could come to his party the next day, and W agreed, and K showed up before time.
(d) L requested R to lend her the microwave for an hour and returned it the next day.

Head Office: 127, Zone II, MP Nagar, Bhopal |+91-7676564400| https://www.toprankers.com Page 30 of 40
SECTION - D: LOGICAL REASONING

Directions (Q.106-Q.135): Read the passage given below and answer the questions that follow-

Passage (Q.106-Q.110): China’s southwestern regions battled fires this week following a severe heat wave,
putting their autumn harvest under pressure. Europe last month experienced heatwaves and wildfires and further
west, the US government has for the first time ever declared a water shortage on the critical Colorado River.
Parts of India experienced a severe heatwave in March, which shrunk the wheat output. True, the southwest
monsoon hasn’t disappointed. With a cumulative rainfall of 71cm till August 24, it’s 9% above normal. And the
storage level in 143 major reservoirs till August 18 was 125% of the last decade’s average water level. At a
regional level, however, rainfall has been deficient across a vast swathe from UP to Bengal. While the gap has
narrowed recently in Bengal, the dry spell in July-August is bound to adversely impact the national paddy output.

There’s a strong link between paddy and water shortage – because it’s a water-guzzling crop. So the message
from climate change in this context is, to get smart about how you use water. The telling statistic is that India
has 17% of the world’s population and 4% of its water resources. Irrigation takes up 90% of the country’s
groundwater draft. And Indian farmers use two to four times more water to produce a unit of grain as compared
to China and Brazil – a terrible waste. This has serious environmental consequences too. A study by Nabard and
ICRIER showed that paddy and sugarcane, which use more than 60% of the irrigation water available, are often
cultivated in the most water-scarce regions.

Clearly, India’s cultivation patterns are out of sync with its resource endowments. And states will have to change
habits. To illustrate, Punjab is unsuitable for paddy, but it also has yielded well above the world average.
Therefore, policies that incentivise farmers in Punjab to move to less water-intensive crops need to be
complemented by higher yields in eastern India. And whatever the policy, it can’t ignore the criticality of price
as a signal to incentivise farmers. Market-responsive prices reflect the scarcity of resources. For example, free
electricity incentivises more borewells, which makes water seem more abundant than it actually is. Allowing
prices to reflect scarcity doesn’t mean governments cannot provide cash subsidies. India, however, can no longer
afford in-kind subsidies that distort prices. Prices work better than exhortations.

106. What can logically be assumed as the root cause of water problems in India?
(a) Shortage of rainfall. (b) Irrigation techniques.
(c) Severe heat waves. (d) Selection of wrong crops.

107. Which of the following is a valid inference from the passage?


(a) India’s cultivation pattern is out of sync with its resource endowments.
(b) The water problem in India will not go easily as only 4% of water resources for 17% of the population.
(c) Indian states have to rethink water-intensive crops.
(d) The Indian government can continue to provide subsidies.

108. What is the best way forward to address the concern highlighted in the passage?
(a) To stop cultivating high water-consuming crops immediately, and import them.
(b) To bring measures to improve the water table.
(c) To increase the price of cereals requiring too much water for irrigation.
(d) To stop all forms of subsidies concerning agriculture.

109. If things remain the same, then the world is probably looking at-
(a) Extinction of some crops. (b) Imminent food scarcity.
(c) Acute shortage of water. (d) High prices of resources with no subsidies.

Head Office: 127, Zone II, MP Nagar, Bhopal |+91-7676564400| https://www.toprankers.com Page 31 of 40
110. The author would most likely object to the fact that -
(a) Shortage of water and heat waves have no connection.
(b) High electric charges can help improve the water table.
(c) Good South West monsoon does not help the paddy farmers of Punjab.
(d) India can learn better irrigation from China and Brazil to save water.

Passage (Q.111-Q.115): No one, not even the British, makes train stations that look like palaces now, and while
comparison in art is a dummy’s game, anyone who has been to the Taj Mahal and CST will not have to think too
much about what is more awe-inspiring. It still does not mean the Mughals were more benign than the British.
Elements which lead to architectural design—what a space stands for, who it is for, whether gargoyles must stick
out of its towers—keep changing. To say that because independent India has not been able to make something
like that makes it deficient, is like saying that Italy is the worse country now because they have never been able
to do anything like the Sistine Chapel or that Egyptians were at their best 5,000 years ago when the Pyramid of
Giza came up.
Was British rule good for India is a question that is impossible to answer, because we will never know what the
alternative could have been. That is just the way time and history work. China and Japan didn’t have colonial
rule, but they got able modern legal systems, railways lines and all the accoutrements that Carlson defines
progress by. Colonialism created political organisation but when the rest of the world is witnessing the
phenomenon of nation-states, why wouldn’t it touch the Indian subcontinent, too, whether or not the British
came? On the other hand, if the British were not here, would India have become an independent nation without
other outside interference? The Portuguese came a century before the British and their first instinct, even when
they were the first player, was to seize political control wherever possible with the same intentions.

111. What is the main idea of the passage?


(a) The colonial rule proved a blessing in disguise for India.
(b) India would have progressed to current levels even without colonization.
(c) The development done by the British could have been done by the Portuguese if they had ruled India.
(d) Colonial rulers do not like resistance from the citizens while developing colonies for their interests.

112. What can be inferred from the passage?


(a) British and Portuguese were not better than each other when treating Indians.
(b) The era of British rule was the golden era of architecture.
(c) Indian architecture did not benefit from the Mughal and British rules.
(d) None of the architectural marvels of the past has been created post the Mughals and the British era.

113. To say that because independent India has not been able to make something like that makes it deficient is like
saying that Italy is the worse country now because they have never been able to do anything like the Sistine
Chapel.’ Which of the following assumption lies behind the argument presented as the question stem?
(a) A cloning of an architectural marvel is not a sign of inadequacy.
(b) A cloning of an architectural marvel is a sign of progress.
(c) The architecture of the contemporary world differs from the past.
(d) India and Italy are analogous regarding architectural styles.

114. ‘Was British rule good for India is a question that is impossible to answer because we will never know what the
alternative could have been.’ The statement is
(a) a fact (b) An inference (c) A judgement (d) A premise

115. What can be inferred about the relationship between the author and Carlson?
(a) They are on the same page. (b) They contradict each other.
(c) One complements the other. (d) It cannot be established in the passage.

Head Office: 127, Zone II, MP Nagar, Bhopal |+91-7676564400| https://www.toprankers.com Page 32 of 40
Passage (Q.116-Q.120): The finance minister last week added heft to RBI’s attack on illegal loan apps. Last
month, RBI issued guidelines that made banks and NBFCs squarely responsible for the conduct of digital
intermediaries hired by them to drum up lending business. GoI has now asked RBI to prepare a ‘whitelist’ of
legal apps, which will be the only ones app stores can host. The scale of the problem can be gauged from data
collected by RBI in 2021. Over two months, January and February, RBI found 1,100 unique Indian loan apps
available in over 81 app stores, of which 600 were illegal.

But this proliferation, much of it a menace, is filling a market hiatus – a lack of financial inclusion on the lending
side. For deposits and transfer payments, digital business models have enabled a rapid scale up of banking
services. There are 464 million beneficiary accounts under Jan Dhan, and the World Bank estimated in 2021 that
78% of Indian adults had a bank account. Banks, however, don’t have the domain expertise to disburse micro
loans on a large scale. This gap is being filled by financial technology firms, which make customer convenience
their USP. Lending apps work by the 2-1-0 formula – two minutes to decide, one minute to transfer and no
human contact and zero collateral. But of course, unregulated by RBI, these lenders often resort to tactics
favoured by loan sharks. Moreover, there’s the danger of these apps being used for money laundering and data
theft.

For the moment, steps taken by GoI and RBI will have an impact. However, the only durable solution is to offer
the convenience of loan apps in a regulated space. RBI needs to balance trade-offs between customer
convenience and protection. It might be instructive to look at lessons of the effort undertaken in 2015 to introduce
differentiated banking licences, which led to the creation of small finance banks and payments banks. Another
set of licences for entities that can disburse micro loans quickly is an idea worth pursuing.

116. What is the most critical message of the author in the passage?
(a) Unregulated lending apps emerged in the market due to the financial exclusion of some from the organised
sector.
(b) Illegal loan apps are a menace, but they indicate a gap in the demand and supply of convenient loans.
(c) RBI has started to rein in the illegal lending apps, which have had enough of a free hand in cheating gullible
customers.
(d) Lending apps have the purpose of providing a convenient loan to anyone and everyone.

117. What can be inferred from the passage?


(a) Loan market has a promising scope in India.
(b) Education and awareness can save customers from the trap of illegal apps.
(c) Improvement in digital technology can lead to convenient financial disbursement.
(d) Loan sharks manoeuvre the banking system.

118. Which of the following is a true statement about micro loans?


(a) It is only required by a few underprivileged.
(b) Government shall arrange to provide easy small-amount loans.
(c) Most of the frauds occur in this loan segment.
(d) Micro loans are small but not easy to get generally including the main banks

119. With the introduction of ‘white list apps’ –


(a) Data theft will stop.
(b) Loans disbursement will be simple.
(c) Prospective customers will be safeguarded from the risk.
(d) RBI can supervise in better ways.

Head Office: 127, Zone II, MP Nagar, Bhopal |+91-7676564400| https://www.toprankers.com Page 33 of 40
120. The author would most likely agree that –
(a) Loan sharks are not easy to deal with their pan-India presence.
(b) Illegal loan apps have a role in the economy till anything legal replaces them.
(c) The unorganised sector of loans can push RBI to new measures.
(d) RBI has come out of slumber while much damage has already been done.

Passage (Q.121-Q.125): A constitution bench is now entrusted with the job of evolving uniform norms for
hearing arguments in those cases where the death penalty is a potential sentencing option. Given the finality of
capital punishment and the many worrying tendencies of lower courts, a code of procedural dos and don’ts will
really help. In fact, Supreme Court’s scrutiny, initially by a three-judge bench, was set off by a trial court giving
the death penalty to a convicted person on the same day the prosecution won the case. Good judicial practice
demands judges consider mitigating circumstances – a traumatic past, mental health – before pronouncing a
death sentence. With the Criminal Procedure Code sketchy on what a meaningful sentencing hearing should
entail, trial court judges getting authoritative SC guidance will help.

India seldom executes death row prisoners, reserving it for truly “rarest of rare” crimes – 26/11 terrorist attacks
in Mumbai or the 2001 attempt to storm Parliament or the Nirbhaya gangrape-murder. Nevertheless, trial courts
hand out dozens of death penalties every year. Laws that prescribe the death penalty range from IPC 302 (murder)
to the anti-terror UAPA to the amended POCSO Act that deals with sexual offences against children. But trial
judges interpret these statutes too liberally, despite high courts confirming only a small fraction of death
sentences.

NLU Delhi’s Project39A reveals 144 death sentences awarded in 2021 against 78 in 2020, and a total of 488
prisoners on death row till December 2021. Meanwhile, HCs confirmed 6 death sentences, commuted to life
imprisonment 25, and acquitted 29. Corresponding numbers for SC were 0, 5 and 4. Note that 33 prisoners on
death row were acquitted on appeal. This is one of the arguments used by those sceptical about keeping capital
punishment in the statutes. Poorer convicts often don’t receive quality legal assistance, and capital punishment
for an innocent is an irretrievable miscarriage of justice.
So SC laying down guidelines for hearing arguments on sentencing is hugely relevant. But SC should note its
guidelines are often ignored by trial courts.

121. Why do lower courts pronounce so many death penalties?


(a) Judges do not consider the cases from all angles.
(b) Law provides the death penalty for those crimes.
(c) Judges relate with the victims of heinous crimes.
(d) The trial judges interpret the statutes prescribing the death penalty too liberally.

122. Why courts should not award the death penalty so liberally?
(a) Human life is precious.
(b) Otherwise, justice is not met.
(c) A chance for improvement is everyone’s right.
(d) Innocents take the brunt of the punishment.

123. ‘Nevertheless, trial courts hand out dozens of death penalties every year.’ The above statement is
(a) An inference
(b) An assumption
(c) A judgement
(d) A fact

Head Office: 127, Zone II, MP Nagar, Bhopal |+91-7676564400| https://www.toprankers.com Page 34 of 40
124. Which of the following weakens the argument that good judicial practice demands judges consider mitigating
circumstances – a traumatic past, mental health – before pronouncing a death sentence?
(a) Some capital crimes are committed in such an emotional state that the perpetrator does not think about the
possible consequences.
(b) Some of those executed may not have been capable of being deterred because of mental illness or defect.
(c) The anticipatory suffering of the criminal, who may be kept on death row for many years, makes the
punishment more severe than just depriving the criminal of life.
(d) Imposition of appropriate punishment is the manner in which the courts respond to society's cry for justice
against criminals.

125. What can be expected after the guidelines of the Supreme Court?
(a) A sudden fall in the number of death penalties.
(b) Status quo will remain.
(c) Lower courts will be more judicious in awarding capital punishments.
(d) Lower courts will represent their case as a challenge.

Passage (Q.126-Q.30): In many indigenous cultures, memory for routes, signs, topography, places and the
names of places is essential for survival. Native Americans, before European colonisation restricted their
territories, were known to have an extraordinary memory for landmarks. Colonel Richard Dodge of the US
Army, who documented the social lives and customs of Native Americans in the second half of the 19th century,
noted that ‘each hill and valley, each rock and clump of bushes, has for him its distinguishing features, which,
once seen, he knows forever after’. As a result, ‘with no knowledge of astronomy, of geography, or of the
compass, the Indian performs feats of travelling for which a white man requires all three’.

The Inuit use the winds like ancient seafarers used the stars, as a natural compass. They develop what
anthropologists call a ‘memory scape’ – a mental map that embodies not just a physical world but a cultural one
too, in which environmental features are charged with cultural and historical meaning. This is not unlike the song
lines of Aboriginal Australians, which describe in great detail the tracks of their ancestors, so that you can find
your way anywhere, taking in waterholes and places of shelter, if you know the right song. Aporta has observed
that even though the Inuit have lived in permanent settlements since the 1960s, they still travel extensively to
hunt. Spatial habits die hard, especially when they are so fundamental to cultural identity.

Great navigators such as the Hiwi, the Inuit and Native Americans are commonly thought to have an instinctive
understanding that tells them which way they are facing. A handful of researchers over the years have even
claimed that humans, like birds, insects and some other mammals, can feel the Earth’s magnetic field.
Disappointingly for some, there is little evidence that humans have ever possessed a directional or magnetic
sense – the fact is, we don’t really need it, because our other senses are more than adequate for our wayfinding
needs, at least when we attend to them. This is what people with a great sense of direction are good at. Their
extra sense is simply the ability to apply all their other senses in careful observation.

Today, for most of us, the wayfinding skills of our ancestors can feel remote, even redundant. All we need to do
to get where we want to go is tap our destination into a smartphone and follow its instructions. Yet having a
sense of place – something a GPS device will never give us – is still important. The need to know where we are,
and to feel safe in our surroundings, is part of the human condition.

Head Office: 127, Zone II, MP Nagar, Bhopal |+91-7676564400| https://www.toprankers.com Page 35 of 40
126. The main objective of the passage is to:
(a) Delineate the navigational skills of indigenous peoples.
(b) Contrast the navigational skills of indigenous communities to those of contemporary humans with
technology access.
(c) Describe how colonialism made indigenous communities lose their navigational skills.
(d) Highlight the various senses used by indigenous peoples for navigation.

127. Which of the following is NOT TRUE according to the passage?


(a) White people have always needed knowledge of astronomy, geography and the compass to help them to
navigate.
(b) The Inuits know the direction of winds intimately and use them as a sort of compass.
(c) Navigational skills are still important for modern people.
(d) The Inuits and the Aboriginal Australians possess similar navigational skills.

128. Which of the following is not required by humans for navigation?


(a) A strong memory.
(b) Applying all senses in careful observation.
(c) A smartphone.
(d) The Earth’s magnetic field.

129. Each of the following CANNOT be inferred from the passage, EXCEPT?
(a) Having wayfinding skills is important for both indigenous and modern societies.
(b) A native American cannot travel for long distances without getting lost.
(c) Inuits know directions because they can sense the earth’s magnetic fields.
(d) For indigenous communities, natural features do not have cultural or historical meanings.

130. The Inuits still travel extensively to hunt because


(a) they are expert hunters.
(b) they are fond of hunting.
(c) it is innate to their way of life.
(d) they have a strong sense of direction and can use winds like a compass during hunting.

Passage (Q.131-Q.135): Eight people in a company were posted at different designations. The designations of
these people in decreasing order were Chairman, CEO, Senior Manager (Sr. Manager), Deputy Manager (DM),
Assistant Deputy Manager (ADM), Manager, PO, Clerk. Four people were ranked between E and F where E
was ranked higher than F. Only one person is ranked between F and H where H is not the manager. Two people
are ranked between G and H. A was ranked immediately higher than D but none of them is CEO. B was ranked
higher than C. A and B does not have consecutive designations.

131. What is the designation of B?


(a) Chairman (b) CEO (c) Sr. Manager (d) ADM

132. How many people were ranked higher than A?


(a) 1 (b) 2 (c) 3 (d) 4

133. What is the designation of F?


(a) PO (b) Manager (c) DM (d) CEO

Head Office: 127, Zone II, MP Nagar, Bhopal |+91-7676564400| https://www.toprankers.com Page 36 of 40
134. If all the people are ranked in the alphabetical order, from higher designation to lower designation, then for how
many people the designation remains same?
(a) 0 (b) 1 (c) 2 (d) 3

135. Who among the following is Clerk?


(a) H (b) G (c) C (d) F

Head Office: 127, Zone II, MP Nagar, Bhopal |+91-7676564400| https://www.toprankers.com Page 37 of 40
SECTION - E: QUANTITATIVE TECHNIQUES

Directions (Q.136-Q.140): Answer the questions based on the information given below.
The given bar graph shows the number of boys and percentage of girls out of the total students who appeared
for an exam in 2012 from different schools.
Number of students who appeared for the exam
= Number of boys + Number of girls

136. Find the average number of girls who appeared for the exam from school A, B and C taken together.
(a) 30 (b) 45 (c) 60 (d) 75

137. The number of girls who appeared in exam from school D is what percent of number of students who appeared
from school B?
(a) 20% (b) 25% (c) 40% (d) 50%

138. Find the ratio of number of students appeared in exam from school B & A.
(a) 1:2 (b) 2:3 (c) 3:4 (d) 5:6

139. Find the difference between number of students who appeared in exam from school C and number of girls who
appeared in exam from school E.
(a) 50 (b) 60 (c) 70 (d) 80

140. The number of students who appeared in the exam from school C in 2015 is 25% more than the number of
students who appeared in the exam from same school in 2012. The number of boys is 50% more than number of
girls appeared in the exam from school C in 2015. Find the difference between number of boys and number of
girls who appeared in the exam from school C in 2015.
(a) 20 (b) 40 (c) 60 (d) 80

Head Office: 127, Zone II, MP Nagar, Bhopal |+91-7676564400| https://www.toprankers.com Page 38 of 40
Directions (Q.141-Q.145): Answer the questions based on the information given below.
The table given below represents the survey statistics of the number of employees who like either tea or coffee,
or both, from five different companies (A, B, C, and D).
Note: Total number of employees in a company = Number of persons who don’t like any of them (tea or coffee)
+ Number of persons who like at least one of them.

Number of Number of
Number of Number of
employees who employees who
Company employees who employees who
like both Tea and don’t like any of
like Tea like Coffee
Coffee them
A 106 98 42 72
B 128 136 58 36
C 78 42 36 24
D 114 84 48 18

141. Find the number of employees in company A.


(a) 228 (b) 234 (c) 238 (d) 244

142. Find the ratio of the number of employees of company C who like only tea to the number of employees of
company D who like only tea.
(a) 9:14 (b) 8:11 (c) 6:13 (d) 7:11

143. Find the difference between the number of employees of company A who like only coffee and the number of
employees of company B who like only coffee.
(a) 20 (b) 28 (c) 26 (d) 22

144. Find the average of number of employees who like both tea and coffee from each of the given company.
(a) 42 (b) 44 (c) 46 (d) 48

145. The number of employees in company D who don’t like either of them (tea or coffee) is what percentage of the
number of employees in company A who don’t like either of them (tea or coffee)?
(a) 20% (b) 25% (c) 27.5% (d) 30%

Head Office: 127, Zone II, MP Nagar, Bhopal |+91-7676564400| https://www.toprankers.com Page 39 of 40
Directions (Q.146-Q.150): Answer the questions based on the information given below.
Different numbers of employees are working in five different companies: A, B, C, D and E in 2017.
1. Number of employees working in A, B, C, D and E in 2017 are in the ratio of 7:5:8:6:4 respectively.
2. Average number of employees working in companies A, C and E in 2017 is 380.
3. Number of male employees working in company A in 2017 is 10% more than the number of female
employees working in the same company in 2017.
4. Number of male employees and female employees working in company B in 2017 are in the ratio 7:8
respectively.
5. Number of male employees working in company C in 2017 is 20 less than the number of female employees
working in the same company in 2017.
6. Number of male employees working in company D in 2017 is 40% less than the number of female employees
working in company C in 2017.
7. Equal numbers of male and female employees are working in company E in 2017.

146. What is the difference between the number of employees working in company B in 2017 and that in company D
in 2017?
(a) 50 (b) 60 (c) 70 (d) 80

147. What is the ratio of the number of male employees working in company B in 2017 to the number of female
employees working in company D in 2017?
(a) 2:3 (b) 2:5 (c) 3:5 (d) 3:7

148. Find the average number of male employees working in all the five companies in 2017.
(a) 166 (b) 168 (c) 170 (d) 172

149. Number of male employees working in company D in 2017 is what percent less/more than the number of
female employees working in company A in 2017?
(a) 10% more (b) 15% more (c) 20% less (d) 25% less

150. Number of male employees and female employees working in company E in 2018 are 20% and 25%
respectively more than the number of male employees and female employees respectively working in it in
2017. Find the number of employees working in company E in 2018.
(a) 294 (b) 298 (c) 302 (d) 306

Head Office: 127, Zone II, MP Nagar, Bhopal |+91-7676564400| https://www.toprankers.com Page 40 of 40

You might also like